You are on page 1of 103

1. 2.

2. (A) The flag is being flown


at half mast.
1 (A) The doctors are drawing some blood. (B) The tool has dug a hole in
the yard.
(B) The scientists are working in the laboratory. (C) The flag marks the hole on
the golf
(C) The students are preparing for an examination. course.
(D) The workers are mixing a cocktail. (D) The pole is holding up the
tent in the
park.
3. 4.

3. (A) It is rainy weather. 4. (A) They are looking at


some
(B) It is a sunny day. documents on the table.
(C) It is snowing. (B) They are wearing helmets.
(D) It is very windy. (C) There are some people
working behind
them.
(D) One of the men is writing
on the
document.
5. 6.
5. (A) There are some building designs on the 6. (A) The man is typing on the
computer.
table. (B) They are both looking at the
laptop.
(B) The woman is drinking a cup of coffee. (C) The men are wearing ties.
(C) The woman is writing a recipe. (D) The men are checking some
(D) The woman is talking on the phone. blueprints.

PART II
DIRECTIONS: You will hear a question or statement and three responses spoken in
English. They will be spoken only one time and will not be printed in your test
book. Select
the best response to the question or statement and mark the letter (A), (B), or
(C) on the
answer sheet.
7. Who are you going to send on the business trip?
(A) I've picked Susan in accounting.
(B) It was a very rewarding trip.
(C) At the start of next year.
8. Why don't we go for a bike ride tomorrow?
(A) I gave Mr. Holland a ride to the airport.
(B) That sounds like fun.
(C) It was 3:30 P.M.
9. Did Monica answer the phone, or was she away from the office?
(A) I'll mark it on the calendar at the office.
(B) Please leave a message.
(C) She was meeting her client at that time.
10. Which theater is the movie showing at?
(A) He's a famous actor.
(B) Well, I'll have to check.
(C) She's over there.
11. Why is there a moving truck parked outside?
(A) We're removing coffee stains.
(B) Into a bigger office.
(C) Because new neighbors are moving in.
12. What should I bring on the camping trip?
(A) You'll need hiking boots.
(B) He's on a business trip with his colleague.
(C) Yes, we should.
13. You will receive five days off next month.
(A) I had a great time at the resort.
(B) I turned the equipment off.
(C) Will it be paid or unpaid?
14. Did Olivia already return the rental car?
(A) Yes, just this morning.
(B) There are several different models.
(C) I'm ready to order now.
15. Isn't this area off limits to motor vehicles?
(A) It's fifty percent off today.
(B) There is a walking path only.
(C) Actually, it's a stolen vehicle.
16. I'd recommend using the stairs today.
(A) Can you tell me why?
(B) No, I didn't stare straight into the camera.
(C) I usually use the copy machine at the corner.
17. When will I receive this month's paycheck?
(A) The conference will be held next month.
(B) Before March 3.
(C) In the bottom drawer.
18. Do we have enough gas to get to the airport?
(A) Who arrived at the airport yesterday?
(B) We don't have to worry about it.
(C) She's the chief flight attendant.
19. Why hasn't the travel itinerary been sent out yet?
(A) At Terminal 6.
(B) He was a travel agent.
(C) We haven't decided on the dates.
20. Who forgot to turn off the lights last night?
(A) We were waiting at the traffic lights.
(B) I'm guessing it was John.
(C) Kelly will take a day off tomorrow.
21. How many copies of this handout do you want made for the meeting?

(A) The report was short.

(B) I’m meeting with the Board of Directors this afternoon.

(C) Twenty-five should be enough.

22. I’ve proofread this book, so I’ m going to mail it.

(A) Wait. I need to take a look first!

(B) Sure, give me 10 minutes.

(C) The post office is on Cordova Road.

23. I’d like an aisle seat , please, towards the front of the plane.
(A) I’m sorry, sir. I only have window seats available.

(B) My back hurts when I sit in these chairs for too long.

(C) I agree. The food was excellent last night.

24. I’d like you to type this letter this afternoon or by tomorrow morning.

(A) OK. I can get it done after lunch.

(B) I tried calling but got a busy signal.

(C) It’s Tuesday tomorrow.

25. Why did you close the branch early?

(A) There was a power outage, so I couldn’t serve customers.

(B) I had to close the branch early.

(C) It’s a long way from here.

26. Who is coming to the mortgage and insurance presentation?

(A) Ian and Brian will be joining you.

(B) I don’t have any insurance.

(C) There’s a schedule of activities on the hall board.

27. Where is the new accountant going to work?

(A) He’ll have an office on the fifth floor.

(B) I’m sure he’ll work the same hours as us.

(C) It’s not an option, I’m afraid.

28. When did you need the budget spreadsheets by?

(A) As soon as possible.

(B) I haven’t made them yet.

(C) It’s three o’clock in London.

29. What is he going to talk about?


(A) He might talk for over two hours.

(B) It says in the program.

(C) I’ve heard he gives very interesting talks.

30. Should I hire Mr. Mathers, or should I hire Mr. Williams?

(A) I’d go for the candidate with the most experience.

(B) Williams is a common family name.

(C) I’m going for lunch.

31. I can’t find the filing cabinet keys anywhere.

(A) It’s getting late.

(B) I hope they’re not locked inside the cabinet.

(C) I’m filing the papers as fast as I can.

PART III
DIRECTIONS: You will hear some conversations between two people. You will be asked
to answer three questions about what the speakers say in each conversation. Select
the best
response to each question and mark the letter (A), (B), (C), or (D) on your answer
sheet.
The conversations will not be printed in your test book and will be spoken only one
time
32. How do the speakers know each other?
(A) They met through a friend.
(B) They take a class together.
(C) They live in the same apartment complex.
(D) They work at the same company.
33. What does the woman suggest that the man do?
(A) Introduce himself to his co-workers
(B) Wear a work uniform
(C) Learn how to make a list of goods
(D) Have a house-warming party
34. What does the man need to do first?
(A) Change his clothes
(B) Attach a name tag
(C) Contact a warehouse supervisor
(D) Read an employee handbook
35. Why is the man calling?
(A) He forgot a document password.
(B) He needs an important document.
(C) He wants to apply for a job.
(D) His computer is not working.
36. When will the woman leave work?
(A) 4:00 P.M.
(B) 5:00 P.M.
(C) 6:00 P.M.
(D) 7:00 P.M.
37. What does the woman suggest the man do?
(A) Extend a warranty
(B) Come to work early tomorrow
(C) Participate in a survey
(D) Check his e-mail

38. Where does the woman work?


(A) At a restaurant
(B) At a university
(C) At a movie theater
(D) At a hotel
39. Why are the tables and chairs currently unavailable?
(A) A shipment has not arrived.
(B) The woman didn't permit their use.
(C) Other people are using them.
(D) The storage room is locked.
40. What does the man clarify?
(A) The expected number of guests
(B) The location of stored supplies
(C) The starting time of an event
(D) The necessary documents

41. Who bought the new computer?


(A) The man
(B) The woman
(C) The man and the woman together
(D) Their employer
42. Why is the man pleased with the computer?
(A) It fits his briefcase.
(B) It is much nicer than his old computer.
(C) He didn’t have to pay for it.
(D) It’s his first computer.
43. How does the woman feel about the computer?
(A) She is looking forward to getting one.
(B) She thinks it looks heavy.
(C) She doesn’t like laptop computers.
(D) She doesn’t think the man deserves it.

44. Where has the woman been?


(A) In Asia
(B) In the office all week
(C) At the conference all week
(D) At home
45. What does the man recommend the woman do?
(A) Start a new project, then check the files
(B) Check the Asian Asian reports immediately
(C) Spend less time out of the office
(D) Attend more meetings
46. What does the woman expect to find in the report?
(A) New proposals
(B) Bad news
(C) A plesamt surprise
(D) Profit figures

47. Where do the man and the woman have to go?


(A) To a lunch appointment
(B) To a meeting
(C) To meet supplier
(D) To a reception
48. Why isn’t the man leaving at the same time as the woman?
(A) He needs to finish reading a letter.
(B) He needs to make some phone calls.
(C) He needs to finish wrting a letter.
(D) He is waiting for a letter to arrive.

49. Why does the woman think their coworkers will be angry?
(A) They dislike the man.
(B) The man will be late again.
(C) They didn’t want the woman to attend.
(D) They have been very irritable recently.

50. What is wrong with the man?


(A) He has lost his job.
(B) He thinks he has the flu.
(C) He wants a raise.
(D) He has a headache.
51. What does the woman imply?
(A) He needs more sleep in order to look good.
(B) He should have stayed at home.
(C) He can recommend a doctor.
(D) He looks better outside of work.
52. Which of the following best describes the situation in the man and the woman’s
office?
(A) They have a lot of deadlines approaching.
(B) They are going through a quiet period.
(C) Everyone is off sick.
(D) The office is closed for the holidays.

53. What happened?


(A) The man loaned his car to the woman.
(B) He left his keys in the parking lot.
(C) He gave his keys to the woman.
(D) His keys were stolen.
54. What is the relationship between the man and the woman?
(A) Siblings
(B) Parents
(C) Spouses
(D) Coworkers
55. Why does the man consider himself lucky?
(A) No one tried to take his car.
(B) He wasn’t harmed when his car was stolen.
(C) It gave him an opportunity to get to know the woman.
(D) He has two cars.

56. How does the man feel about the change Cheryl is making?
(A) He hopes she likes her new job.
(B) He thinks she is making a mistake.
(C) He hopes she likes her new apartment.
(D) He thinks she is a good person.
57. What is Cheryl going to do?
(A) She’s going to coordinate conferences.
(B) She’s going to be a wedding planner.
(C) She’s going to take an extended vacation.
(D) She’s going to move to another city.

58. Assuming it is now April, when will Cheryl start at Beckwith and McDougal?
(A) In two weeks
(B) May 3rd
(C) May 1st
(D) June 1st

59. Where most likely are the speakers?


(A) At an office
(B) At a lawyer's office
(C) At a hardware store
(D) At a local mall
60. What does the man mention about the delivery?
(A) He isn't getting any equipment delivered to the office.
(B) He is getting the small equipment delivered to the office.
(C) He is getting a drill delivered to the office.
(D) He is getting some documents delivered to the office.
61. What does the man say he needs?
(A) An invoice
(B) Some paint
(C) A shovel
(D) Some nails

62. What did the man recently do?


(A) He met with a photographer.
(B) He met with a sales associate.
(C) He met with an interior decorator.
(D) He had lunch.
63. Why does the man want to move the sales desk?
(A) To increase the company's sales
(B) To make it look nicer
(C) To make more room for the woman to work
(D) To give waiting customers more space
64. Look at the graphic. Where will the sales desk be moved to?
(A) Where the help desk is now.
(B) So it is to the right of the entrance.
(C) Where the waiting area is.
(D) They will move the help desk instead.

65. What did the man say about next week?


(A) There will be an inspection.
(B) Some new computers will arrive.
(C) They will have fire drills.
(D) Someone called in sick.
66. Look at the graphic. What department do the speakers work in?
(A) Human Resources
(B) Accounting
(C) Customer Service
(D) Legal
67. What does the woman suggest they do?
(A) Don't say anything
(B) Print out some extra copies
(C) Speak to their supervisor
(D) Put up a sign

68. Where does the man most likely work?


(A) Chemist
(B) Hardware store
(C) Model shop
(D) Medical clinic
69. Look at the graphic. What part is the woman missing?
(A) Decals
(B) Model kit
(C) Snap fit tool
(D) Rubber tires
70. What does the man offer to do?
(A) Deliver it to her
(B) Give her a refund
(C) Cancel the order
(D) Express-post it to her

PART IV
DIRECTIONS: You will hear some talks given by a single speaker. You will be asked
to
answer three questions about what the speaker says in each talk. Select the best
response to
each question and mark the letter (A), (B), (C), or (D) on your answer sheet. The
talks will
not be printed in your test book and will be spoken only one time.

71. Where is this announcement taking place?


(A) In a shopping mall
(B) At a public lecture on recycling
(C) In a class on environmental problems
(D) At a staff meeting
72. Where is the container for newspapers?
(A) On the shelves
(B) Near the copy machines
(C) In a box
(D) Beneath the back windows
73. What time of day is this announcement probably being made?
(A) Early in the morning
(B) Just before noon
(C) Around two in the afternoon
(D) At the end of the working day

74. Who are being introduced?


(A) Two research scientists
(B) Two international business people
(C) Two medical doctors
(D) Two travel agents
75. What is remarkable about Marcia Goldsmith?
(A) She achieved success at a young age.
(B) She is an excellent lecturer.
(C) She lived in one place for 10 years.
(D) She is unmarried.
76. What did David Ireland do for three years?
(A) Worked at UCSF Medical Center
(B) Wrote a book on tropical diseases
(C) Lived in West Africa
(D) Worked for the World Health

77. Which one of these is NOT a sponsor of the festival?


(A) Taos Electronics
(B) Taos Communications
(C) Taos Motors
(D) Taos Telecommunications
78. What is new about the Storyteller Theaters?
(A) They now seat 280.
(B) There are now four of them.
(C) They have new film projectors.
(D) They will not be used in the festival.
79. How much does it cost to see one movie?
(A) $4
(B) $50
(C) $5
(D) $6

80. How long had it rained?


(A) 3 days
(B) 7 days
(C) 2 weeks
(D) A week and a half
81. What advice is given and why?
(A) Shop now because after heavy rains food prices increase.
(B) Keep your hats on because it will rain.
(C) Hold on to your hats because it will be windy.
(D) Keep the skies fresh and clear by driving less.
82. What will the weather be like along the coast today?
(A) Breezy and mild
(B) Sunny and breezy
(C) Sunny with temperatures in the high 70s
(D) Mild temperatures with gusting winds

83. What is the problem?


(A) There are not enough seats available.
(B) There are not enough passengers.
(C) The flight will be delayed for several hours.
(D) Some passengers want an earlier flight
84. What does the airline not want to do?
(A) Bump into passengers
(B) Change the flight time and reprint everyone's tickets
(C) Arbitrarily move passengers to a different flight
(D) Pay people to change flights
85. What does the airline offer?
(A) A later non-stop flight with a better airline
(B) A better class of travel or a free flight
(C) A letter of apology in the future
(D) A later Bight and a free flight coupon.

86. When is it possible to speak to a librarian?


(A) Mondays and Fridays
(B) When the library is open
(C) When you access the library website
(D) On very rare occasions
87. Who should press 6?
(A) Parents
(B) People with hearing problems
(C) People who want to read business magazines
(D) People who can't use the Internet
88. If you want to get information about checking out a video, which number do you
select?
(A) 1
(B) 2
(C) 3
(D) 4

89. What position is being advertised?


(A) Legal assistant
(B) Dental assistant
(C) Foreign coordinator
(D) Bank manager
90. What does the man imply when he asks, "Have you seen the criteria for the
dental
assistant position?"
(A) He is looking at some forms.
(B) He is asking if Julia is familiar with the requirements.
(C) He needs some extra work done.
(D) He wants to learn more about them.
91. Why does the man want to meet the woman?
(A) To teach him the criteria
(B) To make some changes to his office
(C) To sign the contract
(D) To change the criteria

92. Who most likely is the speaker?


(A) A content developer
(B) A secretary
(C) A store manager
(D) A police officer
93. Why most likely is the speaker calling?
(A) To confirm the size on an order
(B) To request some delivery information
(C) To send an extra gift
(D) To purchase a new set of cards
94. Look at the graphic. Who can give the listener planning advice?
(A) John Trizz
(B) Don Trenton
(C) Shubert Mendez
(D) sally Howie

95. Look at the graphic. What is the name of the guest's new album?
(A) Valleys of Fire
(B) Step It Up
(C) Tunnel Vision
(D) Talk Down
96. What does the speaker say influences the guest's music?
(A) Getting married
(B) Moving to America
(C) Moving to London
(D) Meeting Joey Denton
97. What will the guest most likely do next?
(A) Move back to his hometown
(B) Get engaged to his girlfriénd
(C) Release a new album
(D) Get married to his girlfriend

98. Look at the graphic. What floor are they on?


(A) 3rd
(B) 4th
(C) 5th
(D) 6th
99. What is the reason for the celebration?
(A) Introducing a new employee
(B) Mr. Jang's birthday
(C) The retirement of Mr. Jang
(D) A wedding anniversary
100. Who is Mr. Hopkins?
(A) Mr. Jang's nephew
(B) A colleague of Mr. Jang
(C) The owner of the company
(D) A waiter

------------THE END OF THE TEST----------------

READING TEST-002
In the reading test, you will be required to answer several types of reading
comprehension
questions based on a variety of texts. The reading section of the test will last
75 minutes. There
are three parts, and directions are given for each part. You are encouraged to
answer as many
questions as possible within the time allowed.
You must mark your answer on the separated answer sheet. Do not write your answer
in the
test book.
Part 5
Directions: In each section, you will find a word or phrase missing. Four answer
choices are
given below each sentence. You must choose the best answer to complete the
sentence. Then
Rogers
1. the
mark letterTVA,isB,
a service
C, or Davailable
on your answer sheet.to cable customers as part
of the basic service
of Rogers Cable Communications Inc.
(A) exclusively (C) exactly
(B) extremely (D) extensively
2. An old restaurant on 7th Ave. will be rebuilt into the stylish restaurant
Citadela with
a seating for up to 150 guests.
(A) capacity (C) aptitude
(B) intensity (D) preparation
3. The cost of construction as as the construction period has
already
increased twofold over previous estimates.
(A) good (C) fine
(B) well (D) far
4. Please of the Planning staff (Debra Ross @ 222.3926) if you
will or will not be
attending this month’s meeting.
(A) notice (C) deliver
(B) present (D) notify
5. The airline industry also relies on many management, professional, and
administrative support workers to keep operations running .
(A) smooth (C) smoothness
(B) smoothly (D) smoothest
6. According to the manager of ABB Corp., this new service very
positively by
customers who get the service details and cost information quicker.
(A) has (C) have received
received (D) has been receiving
(B) has been
received
7. We cannot provide assurance that manufacturing operations will before
the
end of the year.
(A) retain (C) remain
(B) resolve (D) resume
8. We can understand your strong desire the agreement at the earliest
possible date.
(A) finalize (C) finalizes
(B) to finalize (D) finalized
9. According to the report released last Wednesday, the maintenance staff at PPG
is overworked underpaid.
(A) as (C) nor
(B) and (D) for
10. Those two baseball teams some scheduled games due to the
unexpected hurricane.
(A) should (C) postpones
postpone (D) will postponing
(B) had to be
postponed
11. It is imperative that the contracts at least a month prior to the
beginning of
the project.
(A) finalize (C) be finalized
(B) finalized (D) finalizing
12. Most of the companies have a policy stating that each employee must
a formal,
annual, and written performance appraisal by their manager.
(A) give (C) be giving
(B) be given (D) have given
13. Hotel guests who have some spare time are to take part in the
boat
trip excursion to the nearby island.
(A) encouraging (C) encouraged
(B) encourage (D) encouragement
14. The government released the newest employment figures this
morning.
(A) ever (C) soon
(B) just (D) lately
15. Please make sure you review all the candidates’ names before you
vote, since
it cannot be reversed.
(A) thorough (C) thoroughness
(B) thoroughly (D) most thorough
16. Any products that fail to to the exporter’s specifications are
prohibited
from being exported to that country.
(A) comply (C) abide
(B) conform (D) observe
17. In an effort to attract and clients, advertising and public
relations
services agencies are diversifying their services.
(A) maintainin (C) maintain
g (D) maintains
(B) maintainab
le
18. Employment is to decline 12 percent over the 2010-2020 period due
to
the chronic depression.
(A) expecting (C) decided
(B) projected (D) dedicated
19. The rapid pace of innovation in electronics technology makes for a constant
demand for
and faster products.
(A) newly
(B) new
(C) newer
(D) late
(E)
20. As vehicles have become more , growing numbers of consumers are
reluctant
to make a long-term investment in a new car or truck purchase.
(A) cost (C) expensively
(B) costliness (D) costly
21. Heavy rain caused the of Taylor City’s third annual outdoor arts
and
crafts festival.
(A) cancel (C) cancelled
(B) cancels (D) cancellation
22. Our banquet rooms are capable of events of all types and sizes.
(A) accommo (C) accommodating
date (D) accommodation
(B) accommod
ated
23. The monthly rent cost includes heat and hot water, not
electricity and
cooking gas.
(A) or (C) both
(B) but (D) unless
24. It can often be difficult to real antique furniture from imitation
items.
(A) merge (C) distinguish
(B) suppose (D) expect
25. Upon of your résumé, one of our skilled recruiters will review it
to
determine the most suitable position for you.
(A) receive (C) receipt
(B) receiver (D) received
26. Creating a book of own can be fun and simple with the help of the
Writer-
Pro desktop publishing program.
(A) you (C) yours
(B) your (D) yourself
27. Riverdale Estates is a luxury condominium community that offers an outstanding
of life.
(A) position (C) quality
(B) trade (D) faculty
28. You should become thoroughly with your computer and all of its
software programs before you take an on-line class.
(A) familiarizat (C) familiarity
ion (D) familiar
(B) familiarize
29. The shipment of TK-421 laptops arrived in stores earlier
expected.
(A) on (C) than
(B) from (D) since
30. Most warranties on electronic items do not damage caused by
improper use
of the device.
(A) cover (B) decide
(C) shade
(D) reform
31. Call-Tech Plus offers cell phone service at low monthly rates.
(A) reliable (C) reliably
(B) reliability (D) liableness
32. The car’s trunk is enough to hold luggage for four passengers.
(A) included (C) insufficient
(B) overall (D) spacious
33. The Eagle Room, our largest and most elegant banquet facility, allows groups
of up
to 200 people to enjoy meals in .
(A) comfort (C) comforted
(B) comforts (D) comfortably
34. Telecommuting provides a lot of benefits for the company and
the employee.
(A) every (C) either
(B) both (D) also
35. Our summer interns performed all their tasks even though they
received
only brief training.
(A) capable (C) capability
(B) capably (D) more
capability
36. Hanby Developers specializes in the construction of modest yet comfortable
townhomes that even first-time homebuyers can .
(A) afford (C) require
(B) yield (D) attempt
37. You should not use the Erlian Peak trail you are ready for a
long and fairly
difficult hike.
(A) except (C) unless
(B) rather (D)
notwithstanding
38. Market research reports show that a hotel’s level of is an
important factor
in determining guests’ overall satisfaction.
(A) clean (C) cleanlier
(B) cleanly (D) cleanliness
39. The Liberty Heights Department Store is now proud to offer gift cards in
various
from $25 to $300.
(A) rates (C) amounts
(B) volumes (D) shares
40. with a valid driver’s license or state photo ID may use the city’s
historical
research library.
(A) Anyone (C) Each
(B) Other (D) Whomever

Part 6
Directions: Read the texts that follow. A word or phrase is missing in some of the
sentences.
Four answer choices are given below each of the sentences. Select the best answer
to complete
the text. Then mark the letter A, B, C or D on your answer sheet.
Questions 41-43 refer to the following classified advertisement.
Are you ready to earn some extra cash over the holidays?
Paradise Clothing Store wants part-time store clerks needed to help out during our
busy
holiday shopping season. The positions are perfect for students, parents or anyone
else who
desires in their work schedule. Come and apply today. We offer paid
training, a
41. (A) flexible
(B) flexibly
(C) flexibility
(D) more flexible
competitive pay , and generous employee discounts. But that’s not all.
We offer many
42. (A) weight
(B) value
(C) degree
(D) rate
more benefits these, including free uniforms, sales bonuses, and
regular pay
43. (A) besides
(B) except
(C) until
(D) between
increases. To find out more about the positions, visit our employment Website at
www.paradise.com.
Questions 44-46 refer to the following memo.

INTER OFFICE MEMORANDUM


To: All Operations Department staff
From: Daniel Burnham, Managing Director
Subject: Moving Instructions

I have some good news! Work crews will start our long-awaited office renovation
project next
Tuesday. During the construction, we will move to offices on the 9th floor. After
the upgrades
are completed, our office will have a layout that will give us all easy
access to
equipment.
44. (A) centrally
(B) centralize
(C) centralized
(D) centralization
We do the move on Monday afternoon, and all employees to use the
back exits while
45. (A) requested
(B) requesting
(C) are requested
(D) requests
moving. Later today, you will receive boxes, tape, labels and other for
the move.
46. (A) habits
(B) supplies
(C) sources
(D) factors
We will pack on Monday morning, and later today you will get a seating map for the
new
office. Thank you in advance for your cooperation on this relocation.

Questions 47-49 refer to the following letter

To Whom It May Concern

I've been Joe Employee’s Manager at XYZ Company since 1997, and even promoted Joe
to
the role of Senior Software Engineer last year.

Joe has proven to be a employee with excellent communication skills.


47. (A) reliant
(B) reliable
(C) reliably
(D) variable

He is respected by other members of our team. Joe is a fast learner and


has excellent
problem solving abilities.
48. (A) very
(B) well
(C) much
(D) once
I will be disappointed to lose Joe as an employee, I am happy to
recommend him as a
solid addition to your company. If you'd like more information, please do not
hesitate to
contact me at (555) 555-5555.
49. (A) Because
(B) Despite
(C) For
(D) Although

Sincerely,
John Simpson

Questions 50-52 refer to the following letter.

Dear Mr. Smith:

Just a note to say thank you for giving me the opportunity to interview with your
company for
the position of loss control specialist. I our conversation and feel my
experience and
education would complement
50. (A). enjoy
(B) will enjoy
(C) enjoyed
(D) have been enjoyed
Raymond Kate Associates. I look forward to being able to put my extensive
knowledge and
experience of working with environmental protection agencies to bring about
effective safety
programs to your company.

As we discussed I have enclosed 3 letters of recommendation my


professional
acquaintances.
51. (A) for
(B) as
(C) with
(D) from
, I am also enclosing some of the safety programs I designed so you can
see first hand
the type of work I am capable of performing to bring about the successful safety
programs the
government requires in organizations today.
52. (A) Furthers
(B) Beside
(C) In addition
(D) As well as
Again, thank you for your time. I am looking forward to hearing from you soon.

Sincerely,
Bill J. Doe

Part 7
Directions: In this part you will read a selection of texts, such as magazines and
newspaper
articles, letters, and advertisements. Each text is followed by several questions.
Select the best
answer for each question and mark the letter A, B, C or D on your answer sheet.

Questions 53-54 refer to the following advertisement

Flyte Bakery
All baked goods are handmade on the premises including
 Cakes
 Seasonal fruit pies
 Breads and pastries
 Cookies
 Personalized dessert for anniversaries, weddings, birthdays, and other
special occasions
(Please order at least one week beforehand.)
Catering for corporate events and private parties is now available! To learn more
about our
rates, telephone us during the business hours below.
Monday – Saturday: 6 AM-7 PM, Sunday: 6 AM- 6 PM
Location: 192 South Street, Sydney
Tel: (02) 927206555

53. What item would need to be ordered in advance?


(A) An autumn harvest apple pie
(B) A personalized birthday cake
(C) Whole wheat bread
(D) Chocolate chip cookies
54. For what information are customers directed to call the bakery?
(A) Store hours
(B) Baking recipes
(C) Driving directions
(D) Catering pieces

Questions 55-59 refer to the following letter


Bartel Manufacturing, Inc.
301 San Andreas Street
Oakland, CA 94621

October 11
Daniel Rhee
Home Country Family Foods
175 Calle del Sol
Cupertino, CA 91031

Dear Mr. Rhee:

Thank you for your interest in Bartel’s food packaging technology.

As we discussed in our telephone conversation, Certain Seal is well suited to the


needs of a
mid-to-large-scale food processing company. It is designed to handle large volumes
and can be
customized for almost any size or shape of food product.
Certain Seal also lengthens the shelf life of your products. As you know, one of
the biggest
causes of food spoilage is excess air left in vacuum-sealed packages. Our system
is especially
effective in eliminating this threat. In the September issue of Industry World
magazine,
comparison testing indicated that Certain Seal removed excess air more efficiently
than similar
vacuum packaging systems (such as the Sigillare 100 produced by TYT Industries)
Food quality is ensured further by the packaging material itself. For the Certain
Seal system,
we use an especially durable plastic that protects foods throughout the freezing,
shipping, and
reheating processes.
This is just an overview of some of the benefits of our technology. I will go into
greater detail
in my October presentation at your office. In order to tailor the presentation to
your needs, I
would like to know more about Home Country’s production requirements. Please
contact me
at our main offices in Oakland at (510) 555-0115 between 9 AM and 6 PM.
Sincerely,
Julie Chan
Sales Director

55. What is most likely Ms. Chan’s goal?


(A) To sell a product to Mr. Rhee’s company
(B) To ask Mr. Rhee for marketing advice
(C) To interview Mr. Rhee for Industry World
(D) To apply for a job with Mr. Rhee’s company
56. Where will Ms. Chan and Mr. Rhee meet?
(A) In Home Country’s offices
(B) At the Industry World headquarters
(C) In Bartel’s offices in Oakland
(D) At the TYT Industries production facility
57. In what industry does Mr. Rhee probably work?
(A) Large-scale agriculture
(B) Restaurants and catering
(C) Plastics manufacturing
(D) Food processing
58. What is stated as a benefit of Certain Seal?
(A) Lower production costs
(B) Reliable packaging
(C) Environment friendliness
(D) Ease of disposal
59. What does Ms. Chan ask Mr. Rhee to do?
(A) Decrease production levels
(B) Schedule presentation
(C) Provide additional information
(D) Customize a product for her

Questions 60-62 refer to the following announcement


23rd Annual NOEE Convention

Join more than two thousands elementary school teachers and administrators for the
annual
convention of the National Organization for Elementary Educators (NOEE) in
Seattle,
Washington, April 4-7. Highlights include:
 Keynote address- “Kids These Day” by Amy Fadden, host of the nationally
syndicated
children’s television program Wake Up and Sing
 Video game fair- Meet representatives from Academic Gamer and Learning Curve
Games to discuss how to use educational video and computer games in your
classroom.
 Interview sessions – Bring your resume and meet with human resources
personnel from school districts throughout the country.
All convention activities will take place at the historic Montrose Hotel, which
also offers easy
access to restaurants, shopping, and museums.
Register for the conference or schedule a job interview today! Visit the NOEE
Website at
www.noee.org

60. Where would this announcement most likely be found?


(A) In a tourist brochure about Seattle
(B) In an advertisement for the Montrose Hotel
(C) In a magazine for teachers
(D) In an elementary school textbook
61. What is suggested about the NOEE?
(A) It produces educational children’s games.
(B) It holds a convention every two years.
(C) It sponsors children’s television programs.
(D) It helps its members find jobs.
62. What is indicated about Amy Fadden?
(A) She is a former NOEE president.
(B) She will be a speech at the convention.
(C) She has been a teacher for 23 years.
(D) She has written several books for children.

Questions 63-66 refer to the following notice


Welcome to the Smythe and Lewes team! We look forward to helping you build a
career with
us. We pride ourselves on the professionalism of our employees. Therefore, we
offer the
following tips to help you serve customers better and make your work as productive
as possible.
We specialize in well-made formal and business attire for men and women from
respected
manufactures. We expect employees to wear similar attire at work, and we encourage
you to
wear products from our stores. Therefore, we offer you a 40% discount on all
merchandise
including shoes and accessories at all Smythe and Lewes locations. This will allow
you to
promote our store and, at the same time, to develop a professional wardrobe of
your own.

It will also introduce you to the products and fashions we carry. As you can
imagine, customers
expect Smythe and Lewes employees to be knowledgeable about our inventory. Please
make an
effort to familiarize yourself with it. This task is best reserved for periods
when there are fewer
customers in the store- in the morning for weekday shifts and, for weekend shifts,
at night before
closing. As our inventory changes from week to week, this needs to be an ongoing
process. With
a little effort, you will soon be able to answer questions from our customers
confidently!

63. For whom is the notice intended?


(A) Clothing manufactures
(B) Smythe and Lewes customers
(C) Newly hired employees
(D) Smythe and Lewes executives
64. What is probably NOT sold at Smythe and Lewes store?
(A) Men’s accessories
(B) Business suits
(C) Formal shoes
(D) Athletic apparel

65. What will Smythe and Lewes give to recipients of the notice?
(A) A discount on store products
(B) Sample items from manufacturers
(C) Two breaks during every shift
(D) Free delivery on large orders
66. What are recipients encouraged to do?
(A) Avoid wearing business attire
(B) Study the store’s inventory
(C) Return defective products
(D) Replenish the stock regularly

Questions 67-68 refer to the following classified advertisement.


FOR RENT– 900 square meters of rarely-available commercial space on the first
floor of the
Mosley Building – a landmark high-rise with a nearly 100-year history; an open
floor plan and
large windows make this space ideal for a restaurant or cafe.

GREAT LOCATION– The main entrance to the space is on a very busy part of Wilson
Avenue,
which has lots of vehicle and foot traffic. There are numerous public parking
areas nearby, and
the convention center is only a 5-minute walk away.

TRANSPORTATION– Close to the 141, 146, and 7X express buses as well as the
Downtown
Tourist Loop boarding point.

INCLUDED– Many recent upgrades, including expanded lobby and enhanced public
spaces;
available high-speed Internet connections; 36-month lease includes reduced rent
for the first 6
months; for more information, call Dee Realty at (808) 555-0930.

67. What is indicated about the Mosley Building?


(A) It has several elevators.
(B) It has a large indoor parking lot.
(C) It is in a heavily traveled location.
(D) It opened to the public six months ago.
68. What is NOT mentioned about the space for rent?
(A) It is on the building’s first floor.
(B) It can be accessed from Wilson Avenue.
(C) It has undergone improvements recently.
(D) It is located near a central railway station.

Questions 69-71 refer to the following e-mail.


From: Arturo Santos (SantosFreightInc@mll.com)
To: Ted Munson (Munson@mll.com)
Subject: Your shipping
Date: November 30

Dear Mr. Munson,


I just wanted to follow up on our phone conversation from the 27th and confirm
that I will come
to your residence on Saturday, December 12th with shipping boxes and packing
materials.
Depending on traffic, I expect to arrive at your home at 11 in the morning.

To label your boxes, you will need to do the following: Mark all boxes clearly and
label them
as “one of a total.” So, for example, the first of 29 boxes would be 1/29, the
second would be
2/29, and so on. You should also mark the destination city and country on top of
each box. In
your case, you would write “Chicago, USA.” You will also want to write “Fragile”
and “Top
Load” on the boxes in which you pack your ceramics and other delicate kitchenware.
Remember
that no loose or unpacked items can be accepted by the receiving warehouse in
Chicago.

As we discussed earlier, my crew of movers will meet you at your home at 10 A.M.
on January
15th to pick up your items. The delivery time to the warehouse is normally 4 days.
Please do
feel free to contact me with any further questions you might have.
Regards,
Arturo Santos
69. What is one purpose of the e-mail?
(A) To confirm an appointment
(B) To apologize for shipping damage
(C) To request an extension on a deadline
(D) To describe the contents of a shipment
70. What can be inferred about Mr. Munson?
(A) He is shipping items from Chicago.
(B) He is shipping some easily broken items.
(C) He used to work for a moving company.
(D) He operates a kitchen supply company.
71. When most likely will Mr. Santos’s moving crew visit Mr. Munson?
(A) On November 30
(B) On December 12
(C) On January 15
(D) On January 29

Questions 72-75 refer to the following memo.


MEMORANDUM
To: All employees
From: Yvonne Nguyen, office manager
Subject: The Clean Earth Program

We actively encourage staff participation in our recycling efforts, “The Clean


Earth Program.”
Many among our staff have been involved with the program since it started nearly a
decade ago,
but some newer employees may not be familiar with it. To that end, I’d like to
meet with
everyone this Friday at 5 P.M. in Staff Room B and show you how the program works.
Attendance is mandatory, and it will take about 20 minutes to run through
everything.

Before we meet, there are a few things you should know. We’re very lucky in that
we could
enlist the help of D&S Removal Services for our recycling needs. They have
provided us with
the big recycling container you see beside the photocopier as well as
comprehensive pick-up
service. You may place any recyclable items – from folders to cell phones to old
printers – into
this box for collection. However, we ask that you return used ink cartridges to
our office
manager, Marge Glowacki, for refilling. If you would like a small container to use
by your desk,
you can request one by calling David Lembke in custodial services at X-5822.
Through the Clean Earth program, we are striving to recycle at least 70% of our
office waste.
The program got off the ground when our media room librarian, Douglas Yoon, needed
to
dispose of 200 of our old seminar videotapes. He contacted a friend of his at D&S
Removal
Services, and they immediately set up the convenient pick-up program we have now.
This
means, by the way, that you are welcome to toss your unwanted home videotapes into
the big
collection box for recycling. We hope everyone will do their part in meeting our
recycling goals.

Thank you!
Yvonne Nguyen
Office manager

72. What is one purpose of this memo?


(A) To recognize an employee’s service anniversary
(B) To seek volunteers for an upcoming event
(C) To announce a new recycling program
(D) To give details about a training session
73. The word “enlist” in paragraph 2 is closest in meaning to
(A) secure
(B) summarize
(C) assign
(D) offer
74. Who most likely started the Clean Earth Program?
(A) Yvonne Nguyen
(B) Marge Glowacki
(C) Douglas Yoon
(D) David Lembke
75. Which items are NOT collected beside the photocopier?
(A) folders
(B) videotapes
(C) ink cartridges
(D) cell phones

Questions 76-80 refer to the following information on a Website.


Market Focus Inc. – Be a Product Tester
Home Community Join
Test products! Earn money! Influence business and industry!

Becoming a product tester for Market Focus Inc. is a great way to earn money, make
your
opinions heard, and even pick up insight into the world of market research. Here
are the answers
to Frequently Asked Questions (FAQs) about product testing.
What kind of company are you, and what do you do?
We are a full-service market research firm that does innovative product tests for
a broad range
of clients including such award-winning companies as Hanson Foods. The company was
founded by market research pioneers David and Beverly Clawson nearly 40 years ago.
We were
nearly bought out by the large firm DK Research last November, but David and
Beverly instead
passed Market Focus Inc. down to their sons, Jeff and Daniel. So, for us, market
research
remains a family affair. We are also a proud member of the National Association of
Food
Engineers.
I want to be a product tester. What do I do next?
Once you are registered to become a product tester for Market Focus Inc., your name
will be
placed on an availability list. When you are chosen for market research, you will
be notified by
phone or e-mail. You will then come to our facility on Logan Street to participate.
Depending
on the study, you may test food items, toys, cell phones, or even large appliances.
To sign up
with us, click the “join” link in the upper right and fill out the electronic
registration form. The
information you provide will go directly to our database, so note that we are not
able to accept
e-mailed applications.
I’ve been chosen for a product test. How does it work?
First, our recruiter will inform you how much your test pays. You will then be
given the survey
forms at our first floor reception area, and go to our research room across the
hallway. The 200-
seat eating area of this modern facility also serves as a working restaurant which
is open for
lunch daily from 11 A.M. to 3 P.M. Please note that you may not bring your own food
and drink
into the room, as it is a controlled environment. For the convenience of families,
there is a
children’s playroom next to the research room. Most of our tests last about an
hour.
How often can I participate in product tests?
Names are chosen by a computer at random, based on factors including age and
product
preferences. You might participate in product tests once or twice in a short time
period, and then
several months will pass before your next test. Some people get called more than
others, but at
most you may do the tests 3-4 times a year.
I’m available for a product test. Can I call you and take part in one?
We’re afraid not. Part of market research is having a random group of participants.
But please
do free to call us to confirm that your contact information is up-to-date.

76. What is indicated about Market Focus Inc.?


(A) It has always been run by the same family.
(B) It has won several awards for its service.
(C) It recently built a new headquarters building
(D) It started its own professional organization.
77. How most likely would a person register to be a product tester?
(A) By e-mailing the company
(B) By visiting a different part of the Web site
(C) By visiting the company in person
(D) By phoning the company
78. The words “pick up” in paragraph 1 are closest in meaning to
(A) spread
(B) resume
(C) obtain
(D) lift
79. What is NOT indicated about the research room?
(A) It functions daily as a restaurant.
(B) There is a children’s play area inside.
(C) No outside food is allowed there.
(D) It is located on the first floor.
80. What can be inferred about the company’s product tests?
(A) They mostly involve toys.
(B) They all pay the same amount.
(C) They are done in groups of three.
(D) They cannot be participated in regularly.

Questions 81-85 refer to the following notice and letter.


Peak Global Tours
We make travel fun, exciting, and affordable on tours to countries all over the
world!
Would you like to travel for free?
 Register 15 people for a trip and receive complimentary travel.
 Register 30 people for a trip and receive complimentary travel plus a $100
bonus!
We will
 Supply guidebooks to group members and handle all bookings and paperwork.
 Outfit your group members with luggage tags and name tags.
 Provide a 24-hour hotline for you and your group.
To learn more, call our central office at 1-800-555-0154, or visit our Website at
www.peakglobaltours.com

Peak Global Tours


176 New Bridge Road, Suite 204, Singapore 059413
Tel: 6532-0236
www.peakglobaltours.com

Ms. Se Ying Tan


73 Holland Drive
Singapore 149735

September 19

Dear Ms. Tan:

We are excited that you have elected to be a group leader! We know that you and
the 16
people in your group are going to have a wonderful time on the Peak Global Tours
trip to
Florence and Rome (May 7- May 18). Feel free to add more people to your group at
any time
up to four weeks before the trip; however, anyone who books a trip after December
31 will
have to pay additional fees.
We will email periodic updates about the trip to you. In the meantime, please
consider joining
the group leaders’ forum on our Website to receive valuable travel trips. We will
mail you and your group members a departure package with your final itinerary and
flight
information approximately 10 days before your scheduled departure date.
With best wishes,
Tony Sim

81. What is the purpose of the notice?


(A) To provide information about trends in international travel
(B) To announce the launch of a new travel Website
(C) To list services provided by a travel agency
(D) To advertise an upcoming tour of Italy

82. What is indicated about Peak Global Tours?


(A) Its staff can be contacted at any time.
(B) Its provides uniforms to its group leaders.
(C) It sells luggage and travel accessories.
(D) It specializes in travel to Europe.
83. What is suggested about Ms. Tan?
(A) She will be asked to pay additional fees.
(B) She is eligible to receive a fee trip.
(C) She has sent a package to Tony Sim.
(D) She has posted travel information on a Website.
84. In the letter, the word “tips” in paragraph 2, line 2, is closest in
meaning to
(A) pieces of advice
(B) pointed ends
(C) Web pages
(D) gifts of money
85. According to the letter, what will most likely happen in April?
(A) Ms. Tan will travel to Florence and Rome.
(B) Ms. Tan will book another trip.
(C) Peak Global Tours will send some documents to Ms. Tan.
(D) Peak Global Tours will add people to Ms. Tan’s group.

Questions 86-90 refer to the following emails


From: eewiese@office-vpbp.de
To: jbeck@reyna-ibs.org
Subject: Volume out
Date: July 25
Dear Mr. Beck,

The proceedings of the February Small Business Solutions Conference in San Antonio
will be
published in August, not in September as originally planned. This means that I will
soon be
sending out five copies of the volume free of charge to all contributors, and I
need to know
where your copies should be sent. The address I have on file is:
Jason Beck
Reyna Institute for Business Studies
2300 Roepen Dr.
Irving, TX 75039
Is this address correct? Also, for your information, you may order ten additional
copies of the
volume from the publisher with a 20% discount for contributing authors. If you want
to do so,
please contact
Peter Bauer Publishing
Generalsweg 13
10785 Berlin

Thank you once again for your contribution to the proceedings. It was a pleasure
working with
you.

Elena Wiese
Volume Editor
From: jeck@reyna-ibs.org
To: eewiese@office-vpbp.de
Subject: Re: Volume out
Date: July 26
Dear Ms. Wiese,

Thank you for your email. I am delighted to hear that the volume will come out
sooner than
planned.
Concerning your question about my current address, actually we just moved to a new
building
last week. It’s only a few miles always, but we are no longer in Irving. My new
address is:
Jason Beck
Reyna Institute for Business Studies
391 Parker Ave.
Euless, TX 75049

Thank you,
Jason Beck
86. Who is Mr. Beck?
(A) The editor of a journal
(B) A bookstore owner
(C) A graphic designer
(D) A contributor to a publication
87. What does Ms. Wiese ask Mr. Beck to do?
(A) Confirm his contact information
(B) Approve corrections to an article
(C) Send her a list of his publications
(D) Make a presentation at conference
88. How many copies of the publication will Mr. Beck receive from Ms. Wiese?
(A) One
(B) Five
(C) Ten
(D) Fifteen
89. Where will Ms. Wiese send the copies?
(A) To San Antonio
(B) To Berlin
(C) To Irving
(D) To Euless
90. When did Mr. Beck move to a new office?
(A) In February
(B) In July
(C) In August
(D) In September

Questions 91—95 refer to the following notice and letter.


Obtaining Certified Vital Records

You must apply in person or by mail for certified copies of vital records, such as
birth
certificates, marriage licenses, or health records. Our office does not accept
applications by
telephone, fax, or through our Web site. You must provide the following
information with
your request:
• Name
• Address
• Type of record requested
• Passport, government ID card, or other photo
identification (Photocopies of these documents are
acceptable)
The charge is $15.00 per copy issued. We accept checks, money orders, and credit
cards.
In Person:
You can apply in person at 5 East Amber Street, Room 115. The Vital Records Office
is open
8:00 A.M. to 4:30 P.M. Monday to Wednesday; 8:00 A.M. to 5:30 P.M. Thursday; and
8:00
A.M. to 4:00 P.M. on Friday.
By Mail:
Applications for copies of vital records should be mailed to the following
address:
Vital Records, RO. Box 349, Madison, WI 53702. Please include an addressed,
stamped,
business-size envelope with your request. Copies are normally mailed within
fourteen business
days. If you require faster service, you may request our expedited service.
Expedited requests
are normally processed within 24 hours and require an additional charge of $10 per
request,
for a total of $25 per copy.
Requests for expedited service should be mailed to the following address:
Vital Records-Rush Service, PO. Box 567, Madison, WI 53702.

2235 Meyer Way


Appleton, Wisconsin 52206

Dear Sir or Madam:

This letter is to request a certified copy of a marriage license issued last month
in Milwaukee
County to John Allen Heinrich and Elizabeth Ann Miller. I have enclosed a check
for $25, as
well as a copy of my passport. Please mail the certificate to John Heinrich, 2235
Meyer Way,
Appleton, WI 52206.
Thank you.
Yours sincerely,
John Heinrich
91. What is the main purpose of the notice?
(A) To announce a recent price increase
(B) To provide directions to an office
(C) To explain how to obtain certain official records
(D) To describe a new government facility
92. What information must accompany each request?
(A) Identification that includes a photograph
(B) Two copies of recent tax returns
(C) The applicant’s telephone number
(D) A letter written on business stationery

93. On what day does the Vital Records Office close at 5:30?
(A) Monday
(B) Tuesday
(C) Wednesday
(D) Thursday
94. What does John Heinrich request a copy of?
(A) A birth certificate
(B) A marriage license
(C) A passport
(D) A health record
95. Why does John Heinrich enclose a check for $25?
(A) He wants two copies of a document.
(B) He is requesting a very old record.
(C) He is asking to have his records sent to an overseas address.
(D) He wants to receive a document quickly.

Questions 96—100 refer to the following newspaper articles


Cezanta’s Move

Atlanta (United News Service)—Atlanta-based Cezanta Air, the nation’s number four
airline,
announced today that it will cut its domestic airfares significantly. The company
is hoping that
by offering its new promotional fare plan, the carrier will be able to increase
its customer base
and win a greater share of the market. The company promises to charge no more than
$300 for
a one-way economy ticket or $500 for a one-way first-class ticket. The new plan
also includes
the reduction of various ticketing fees and the elimination of several less-
traveled routes at the
end of this year. Cezanta hopes that these changes will raise revenue over the
long term. The
company is also planning to update its airplanes’ interiors and redesign flight
attendants’
uniforms.

Buford Valley Daily News


WILL NEW AIRLINE STRATEGY FLY?
by Ken Daly
Faced with weak revenues, rising labor costs, aggressive pricing from small,
discount
competitors, and most notably high fuel costs, several major air carriers have
struggled
mightily in the past three years. They have experimented with a wide variety of
marketing
strategies without much success. Recently, Cezanta cut its ticket prices by
nearly 50 percent.
However, it is unlikely that this strategy will result in a major increase in
profits for the
airline, as most of the other large airlines will probably cut their prices to
follow suit. Some
analysts are actually expecting Cezanta’s revenues to fall in the upcoming year.
With fuel
prices expected to continue rising and with the cost of fuel accounting for
nearly 40 percent of
all carriers’ operating expenses, any increase in passenger traffic is unlikely
to offset the
decrease in ticket prices. The one sure thing is that, after December, consumers
flying out of
Buford Valley will no longer be able to take advantage of Cezanta’s price cuts.
96. What is suggested about the airline industry?
(A) Operating costs have recently gone down.
(B) Smaller airlines often offer very low prices.
(C) Ticketing fees will be eliminated.
(D) Airlines have increased the amount spent on advertising.
97. What does Cezanta Air plan to do?
(A) Reduce the number of its international flights
(B) Expand its service on less-traveled routes
(C) Eliminate business-class and first-class seating
(D) Modernize the inside of its aircraft
98. What does Mr. Daly suggest?
(A) Other airlines will try to match Cezanta’s prices.
(B) Smaller airlines will be bought by their larger competitors.
(C) Cezanta’s revenues will increase right away.
(D) Passengers’ opinions influence aircraft design.
99. According to analysts, what is the industry’s greatest challenge?
(A) Outdated equipment
(B) Rising fuel costs
(C) Customer satisfaction
(D) Improving safety
100. What can be inferred about Buford Valley?
(A) It ¡s close to Atlanta
(B) It is the site of a new airport.
(C) Cezanta will move its headquarters there.
(D) Cezanta will not fly there next year.

THE END

READING TEST 1
In the Reading test, you will read a variety of texts and answer several different
types of reading
comprehension questions. The entire Reading test will last 75 minutes. There are
three parts, and
directions are given for each part. You are encouraged to answer as many questions
as possible
within the time allowed.
You must mark your answers on the separate answer sheet. Do not write your answers
in your test
book.
Part 5
Directions: A word or phrase is missing in each of the sentences below. Four answer
choices are
given below each sentence. Select the best answer to complete the sentence. Then
mark the letter
(A), (B), (C), or (D) on your answer sheet.
1. An interview with author Tito Flores about ____Adj___ new book will be
broadcast
tonight.
A. himself
B. him
C. his (của anh ấy)
D. he
2. Perend Trail’s new hiking boots will be available in brown _______ black
leather.
A. nor
B. yet
C. and
D. so
3. Mr. Ruotolo’s ___N____ on the new tax changes is scheduled for 10:00 A.M.
A. present
B. presented
C. presentable
D. presentation (bài thuyết trình)
4. Boyd Street Market is Mapleton’s largest retailer of foods ___from___
around the
world.
A. toward
B. from (around the world: từ khắp nơi trên thế giới)
C. above
D. plus
5. Mr. Johansson ___Adv____ accepted the job offer he received from Saco Bike
Works.
A. quicken (v)
B. quickly (adv)
C. quicker (adj)
D. quickness (n)
6. To expand its global reach, Amity Spas will ___Adv____ open its franchise
opportunities
to international prospects.
A. soon (sớm)
B. almost
C. recently
D. already
7. The second-generation XR1280 unit is ___Adj____ to its predecessor, except
for its
reduced weight.
A. equally
B. equal (ngang bằng)
C. equals
D. to equal
8. Zarmeni Mining has been evaluating the benefits of building a mine at the
___Adj____
site.
A. proposed (được đề xuất)
B. structured
C. unlimited
D. educated
9. Vice President Ramos will not make ___any____ decisions until more feedback
has been
gathered.
A. whether
B. what
C. over
D. any
10. If you have ordered more than two items (sản phẩm), be aware they may
arrive in separate
(riêng biệt)__N_____.
A. payments
B. sequences
C. packages (bưu kiện, bưu phẩm)
D. receipts
11. Candidates for the open position must have good interpersonal skills (kỹ
năng liên nhân vị)
and ___N____ working with clients (khách hàng).
A. experience (kinh nghiệm)
B. experienced
C. experiencing
D. to experience
12. Chong Kim was ___Adv____ recommended for the position of merchandise
manager for
Corbin Furniture Mart.
A. thickly
B. currently
C. securely
D. highly
13. Once the returned item is received, a refund will appear on your credit-
card statement
_______ five business days.
A. Within (trong vòng)
B. during
C. since
D. when
14. The printer on the second floor will be out of (hết/ngưng) _service ______
until the
technician arrives on Friday.
A. purpose
B. variety
C. service (dịch vụ)
D. repair
15. Sales of our computer software were good last quarter, but sales for our
mobile applications
have been even ___Adj____ .
A. strong
B. stronger
C. strongly
D. strongest
16. Upon request, the guests at Olane Hotel will be provided vouchers
____for___ free
parking (Phiếu đậu xe miễn phí).
A. on
B. to
C. with
D. for
17. Please review the projected sales figures in the spreadsheets that __V(số
nhiều)_____ to the
e-mail.
A. is attaching
B. had attached
C. attachment
D. are attached (được đính kèm)
18. During tourist season, selling handmade crafts (hàng thủ công) is a
____Adj___ source of
income for local residents.
A. contented
B. dependable (đáng tin cậy)
C. flavorful
D. patient
19. Everyone at the annual Tirnaco exposition seemed __Adj_____ by the new
products on
display.
A. excite
B. excitement
C. excited (phấn khích)
D. excitedly
20. This Saturday, Ritesense customers will have the opportunity to sample a
__N_____ of
Health Bar products.
A. nutrition
B. selection (sự lựa chọn)
C. placement
D. management
21. Ms. Wong has expressed (bày tỏ)___N____ in leading the city’s planned
beautification
project.
A. interest (quan tâm)
B. interests
C. interesting
D. interestingly
22. Mr. Choo was chosen to head the committee for consumer protection (bảo vệ
người tiêu
dùng) from a __Adj_____ pool (một số) of candidates.
A. sizable (đáng kể, nhiều)
B. practiced
C. consecutive
D. missing
23. Please save spreadsheets periodically when updating them to prevent data
from
___V ____.
ing

A. is lost
B. lost
C. being lost (bị mất)
D. losing
24. _______ Gyoh Company’s marketing push, new orders for cash registers (máy
tính tiền)
decreased slightly in the third quarter.
A. As (+ Cl)
B. If (+ Cl)
C. However (+ Cl)

D. Despite (+ N: Mặc dù)


25. All members of the sales team must attend next Thursday’s meeting so that
_______ can
see the sales forecast presentation (Phần thuyết trình dự báo về bán hàng).
A. one
B. someone
C. everyone
D. either one
26. Creating a book of ___Adj____ own can be fun and simple with the help of
the Writer-Pro
desktop publishing program.
(A) you
(B) your (adj: của bạn)
(C) yours
(D) yourself
27. Riverdale Estates (Cty Bất động sản Riverdale) is a luxury condominium
community that
offers an outstanding ____N___ of life (chất lượng cuộc sống).
(A) position
(B) trade
(C) quality (chất lượng)
(D) faculty
28. You should become thoroughly (hoàn toàn: fully)____Adj___ with your
computer and all
of its software programs before you take an on-line class.
(A) familiarization (n)
(B) familiarize (v)
(C) familiarity (n)
(D) familiar (adj)
29. The shipment of TK-421 laptops arrived in stores earlier ____than___
expected (dự kiến).
(A) on
(B) from
(C) than
(D) since
30. Most warranties (bảo hành) on electronic items do not ___V____ damage (hư
hại) caused
by improper use of the device (thiết bị).
(A) cover (bao quát)
(B) decide
(C) shade
(D) reform
31. Call-Tech Plus offers ___Adj____ cell phone service at low monthly rates
(mức thuê bao).
(A) reliable (adj): tốt/ đáng tin cậy
(B) reliability (n)
(C) reliably (adv)
(D) liableness (n)
32. The car’s trunk (cốp xe) is __Adj_____ enough to hold luggage (hành lý)
for four
passengers.
(A) included
(B) overall
(C) insufficient
(D) spacious (adj: rộng)
33. The Eagle Room, our largest and most elegant banquet facility, allows
groups of up to 200
people to enjoy meals in __N_____.
(A) comfort (sự thoải mái, tiện nghi)
(B) comforts
(C) comforted
(D) comfortably
34. Telecommuting provides a lot of benefits for ___both____ the company and
the employee.
(A) every
(B) both
(C) either
(D) also
35. Our summer interns (thực tập sinh) performed (thực hiện) all their tasks
___Adv____ even
though they received only brief training.
(A) capable
(B) capably (adv: có năng lực chuyên môn)
(C) capability
(D) more capability
36. Hanby Developers specializes in (chuyên về) the construction of modest yet
comfortable
townhomes that even first-time homebuyers (người mua nhà lần đầu) can ___V____.
(A) afford (v: đủ năng lực tài chính để mua/thuê)
(B) yield
(C) require
(D) attempt
37. You should not use the Erlian Peak trail (đường mòn của ngọn Erlian) _______
you are
ready for a long and fairly difficult hike.
(A) except (+N)
(B) rather (+N)
(C) unless (+Cl)
(D) notwithstanding
38. Market research reports show that a hotel’s level of ____N___ is an
important factor in
determining guests’ overall satisfaction.
(A) clean
(B) cleanly
(C) cleanlier
(D) cleanliness (sự sạch sẽ)
39. The Liberty Heights Department Store is now proud to offer gift cards in
various
__N_____ (phiếu quà tặng ở các mức khác nhau) from $25 to $300.
(A) rates (n: mức giá)
(B) volumes
(C) amounts
(D) shares
40. _______ with a valid driver’s license or state photo ID may use the city’s
historical
research library.
(A) Anyone
(B) Other
(C) Each
(D) Whomever

Part 6
Directions: Read the texts that follow. A word, phrase, or sentence is missing in
parts of each text.
Four answer choices for each question are given below the text. Select the best
answer to complete
the text. Then mark the letter (A), (B), (C), or (D) on your answer sheet.

Questions 41-44 refer to the following notice.


This notice (41)___V(số ít)____ your reservation for two double rooms, with check-
in on Sunday,
March 5, and checkout on Thursday, March 9. I see here that you have a special
request for one
extra set of blankets and two extra pillows in each room; please note (lưu ý)
(42)__that_____ the
items will be placed in each room, on top of the dresser. There will be no extra
charge for this
request.
(43)___Adv____, check-in time is at 3:00 P.M., and checkout is at noon. Some people
wish to
check in at an earlier time or check out at a later time. (44)_______. This will
help us ensure that
we can accommodate your scheduling needs. We look forward to having you stay with
us.

41. (A) will be 43. (A) Apparently


confirmed (B) As a reminder (xin nhắc là)
(B) confirms (xác (C) In an emergency
nhận) (D) However
(C) is a
confirmation
(D) confirm 44. (A) If these days suit you, please let us know so we can
reserve a shuttle
for you.
(B) For example, some people need a reservation with all
meals included.
42. (A) which (C) If this is your case, please give us a call 24 hours in
advance of your
(B) what arrival. (nếu trường hợp này đúng với bạn, xin hãy gọi cho
chúng tôi 24
(C) these tiếng trước giờ nhận phòng)
(D) that (D) You can always request a wake-up call by contacting the
receptionist.

Questions 45-48 refer to the following e-mail.


To: claimscenter@cheapsure.com
From: lgreenberg@cheapsure.com
Date: November 15
Re: Updates to Office Layout

Dear Claims Specialists:


On January 1, Cheapsure will begin offering homeowners insurance as well as
automobile
insurance. This exciting (45)__N_____ will require a surge in hiring and
adjustments to the layout
of our office space. Tomorrow, I will hold a meeting at 2:00 P.M. in the main
conference room to
discuss the (46)__Adj_____ changes. We have (47)___Adj____ flexibility (sự linh
hoạt) with the
floor plan and would like your input. (48)_______. .

Sincerely,
Lee Greenberg Facilities Administrator

45. (A) performance 47. (A) some


(B) merger (B) each
(C) relocation (C) overly
(D) addition (thêm) (D) very
46. (A) necessarily (adv) 48. (A) Some current employees will switch
departments.
(B) necessity (n) (B) Attached is a map of our new desk assignments.
(C) necessary (adj: cần thiết) (C) Your attendance is thus strongly encouraged.
(Rất mong sự
(D) necessities (n) tham dự của quý vị)
(D) Productivity is expected to double afterward.

Questions 49-52 refer to the following instructions.


Before making travel arrangements, all Saffler Bank employees (49)__V_____
authorization (sự
cho phép). Only after approval has been granted can travel plans be made. To
request
authorization, fill out the first side of the Travel Reimbursement Form. Here you
will provide the
reason for your (50)___N____. Next, submit the form (51)___Pre____ an estimate (nộp
đơn cùng
với dự trù kinh phí) for the cost of travel to your destination. Upon returning, do
not forget to fill
out the second side of the form, where you will report mileage and expenses.
Inclusion of receipts
is required. Expenses for which you cannot show a receipt, such as parking meters
and tips, are
also reimbursable (được hoàn lại). (52)_______.
49. (A) will receive 51. (A) along with (cùng với0
(B) must receive (B) taken from
(C) had received (C) according to
(D) are receiving (D) in the event of

50. (A) trip 52. (A) It is customary to leave a tip.


(B) decision (B) Parking is limited, so arrive early.
(C) situation (C) These expenses should be supported with a written
statement.(Những
(D) appointment khoản chi này cần được liệt kê ra bằng văn bản)
(D) However, routine travel does not require managerial
approval.

Part 7
Directions: In this part you will read a selection of texts, such as magazine and
newspaper articles,
e-mails, and instant messages. Each text or set of texts is followed by several
questions. Select the
best answer for each question and mark the letter (A), (B), (C), or (D) on your
answer sheet.

Questions 53-54 refer to the following contract.


Pinnacle Sports Club Membership Agreement
Member Name: Mary Swansone
Membership Type: 12 months
Begin Date: September 5
Enrollment Fee: $25
Monthly Dues: $32 per month
Payment Method: Credit card
Thank you for joining Pinnacle Sports Club. As a club member, you have unlimited
access to all
gym equipment, fitness classes, and the swimming pool. Please present your
membership card to
the front-desk attendant upon entry. If for any reason you need to discontinue your
membership
before the 12-month contract period has expired, you must write a letter and send
it by mail to the
Pinnacle Sports Club, 171 Aqua street, Germantown, California 95913. A penalty will
apply.
Signed: Mary Swansone

53. What is suggested about Ms. Swansone?


(A) She recently moved to California.
(B) She has not paid her enrollment fee.
(C) She is an instructor in a fitness class.
(D) She has entered into a one-year contract. (Cô ấy mới kỹ hợp đồng 1 năm)
54. Why would Ms. Swansone be required to submit a letter? (Tại sao Bà Swansone đc
yêu cầu
nộp thư?
(A) To cancel her membership (hủy thẻ hội viên)
(B) To gain access to special equipment
(C) To pay with a different credit card
(D) To receive a replacement membership card

Questions 55-56 refer to the following text-message chain.


Harry Matthews (11:19 A.M.) Hi Linhan. Did you already e-mail me the list of this
week’s
invoices?
Linhan Xu (11:24 A.M.) Yes. I sent it over earlier today with the sales-calls
records.
Harry Matthews (11:25 A.M.) Really? I hope I didn’t delete it. Did you accidently
send the e-
mail to Harvey Mattson?
Linhan Xu (11:28 A.M.) Oh, wait - it looks like I forgot to attach it! I’ll send it
over now.
Harry Matthews (11:30 A.M.) I just got it. Thanks!

55. What is the purpose of the text-message chain? (Mục đích của chuỗi tin nhắn là
gì?
(A) To determine whether information has been sent (Để biết xem liệu thông tin đã
được gửi hay
chưa?)
(B) To understand why a decision has changed
(C) To provide instructions on deleting a file
(D) To decide who will deliver a presentation
56. At 11:25 A.M., what does Mr. Matthews imply when he writes, “Really?”
(A) He needs the documents right away.
(B) He deleted the sales-calls records.
(C) He cannot find the attachment. (Ông vẫn không thể tìm thấy file đính kèm).
(D) He sent the attachment to Mr. Mattson in error.

Questions 57-58 refer to the following email.


From: jovanie.blum@messickfmc.com
To: asvoboda@ nyzcomputing.com
Date: 5 May
Subject: Financial consulting

Dear Mr. Svoboda:


My name is Jovanie Blum, and I am a consultant with Messick Financial Management
Company.
My company specializes in international funds, and we have experts on tax law in
France and
Germany. As your computer firm expands its operations into Germany, our experts can
provide
valuable advice to your employees who will be sent to Berlin.
To find out more about what we can do for your employees, please contact me at your
earliest
convenience at 1-267-555-0184.

Sincerely,
Jovanie Blum

57. Why does Ms. Blum most likely want to meet with Mr. Svoboda? Tại sao Blum lại
muốn gặp
Svoboda?)
(A) To apply for a job in Berlin
(B) To sell her company’s services(Để bán dịch vụ của công ty)
(C) To discuss business opportunities in Asia
(D) To review changes in a city’s tax code
58. How could Messick Financial Management help Mr. Svoboda’s company?
(A) By providing information about the German computer industry
(B) By recruiting employees to work in Europe
(C) By recommending international travel services
(D) By giving financial advice to his company’s staff (Cung cấp tư vấn tài chính
cho công ty của
ông ấy)

Questions 59-61 refer to the following customer review.


https://www.therightvenue.com.au/customer_reviews -

Customer Reviews
★★★★✰
12 April
Elena Patterson
My company recently held a banquet (tiệc) at the Calla Courtyard, and it was the
perfect venue for
our event. Initially, I was hesitant to book the space because it had only recently
opened to the
public and there were no customer reviews yet. — [1] — . I decided to give it a try
anyway.
Although it was a little expensive, the setting (khung cảnh) was absolutely
stunning (hết sức đẹp).
— [2] — . The Calla Courtyard has a magnificent view of the bay. The staff had
arranged the seats
in the hall so that the guests could view the sunset through the large glass
windows.
One thing to note is that while the facility does have a kitchen available for use,
it is quite small
and not well-stocked (Điều lưu ý là trong khi có nhà bếp nhưng khá nhỏ và thiếu
dụng cụ. Vì lý do
này…). — [3] — .
All in all, I was pleased with my decision and would definitely consider this place
again for future
events. — [4] — .
59. What is indicated about the Calla Courtyard? (Điều nào sau đây là đúng về Calla
Courtyard?)
(A) It is a relatively new rental space. (là địa điểm cho thuê tương đối mới)
(B) It was recently renovated.
(C) It is a popular venue for business events.
(D) It features an outdoor seating area.
60. What was Ms. Patterson particularly impressed by? (Bà P đặc biệt ấn tượng về
điều gì?)
(A) The friendliness of the staff
(B) The low cost of the rental
(C) The seating capacity of the hall
(D) The beauty of the surroundings (Cảnh quan xung quanh đẹp)
61. In which of the positions marked [1], [2], [3], and [4] does the following
sentence best belong?
“For this reason, it might be wise to hire a full-service catering company.” (Vì lý
do này, điều nên
làm là thuê công ty ẩm thực chuyên nghiệp)
(A) [1]
(B) [2]
(C) [3]
(D) [4]

Questions 62-64 refer to the following postcard.

Hartridge University School of Business


Dear Ms. Wu:

As the Hartridge University School of Business prepares the 100 -anniversary


edition of the
th

Alumni Directory, we are reviewing the information we have on file for all
graduates of our
programs (Chúng tôi đang rà soát thông tin chúng tôi có trong hồ sơ về tất cả cựu
sinh viên của các
chương trình học). We need your assistance in checking the accuracy of the records
we have for
you (Chúng tôi cần bà giúp đỡ kểm tra độ chính xác của thông tin chúng tôi có về
bà). We want to
be certain that your biography, career summary, and contact information are up-to-
date.
Please call us at 207-555-0125 between 9 A.M. and 9 P.M. Monday through Friday. At
the voice
prompt, enter this unique number: 293883 (sau tiếng bíp, hãy nhập dãy số 293883). A
representative will then go over your file with you and record any corrections or
updates that you
give.
Note that we need to hear from you by March 31. Thank you for your time and
attention.

Jutta Verhoeven
Director of Alumni Relations

62. Why is Ms. Verhoeven contacting Ms. Wu?


(A) To issue an invitation to an anniversary party
(B) To request verification of some information (Xin xác minh thông tin)
(C) To provide advice on a career opportunity
(D) To offer a discount on a publication
63. What does the postcard indicate about Ms. Wu? (Bưu thiếp nói gì về bà Wu?)
(A) She teaches at Hartridge University.
(B) She is writing a biography of a business leader.
(C) She plans to make a donation to the alumni association.
(D) She is a graduate of Hartridge’s business school. (Là cựu sinh viên của khoa
kinh doanh, đh
Hartridge.)
64. According to the postcard, what must Ms. Wu provide at the start of the call?
(A) A new phone number
(B) An old account password
(C) A personal identification number (Mã số nhận diện cá nhận)
(D) A payment confirmation code

Questions 65-68 refer to the following online chat discussion.


Mario Lizzardi (11:09 A.M.) Team, sorry to bring this up early on a Monday, but the
due date to
submit our end-of-year report is coming up. We should aim to have each of our
sections done by
Wednesday so we can put everything together before the Friday deadline. I am almost
done with
the information for the technology section (Tôi sắp làm xong phần thông tin cho mục
công nghệ).
Karthik Durav (11:14A.M.) I have already written up descriptions of the program’s
major
accomplishments. I just need to add the number of participants and organizers
involved in each. It
won’t take very long.
Paola Rossi (11:15 A.M.) I have finished the Future Goals section. I still need to
finish the
Predicted Budget Needs section, but I’d like to consult with some of you first.
Mario Lizzardi (11:15 A.M.) I can’t help you with that. I don’t have access to that
information
(Tôi không được truy cập vào thông tin đó) —only you and Human Resources have
access to the
participant lists.
Karthik Durav (11:17 A.M.) Don’t worry, Mario. Paola, I could help to outline
budget needs
with you tomorrow (Tôi có thể giúp cô xây dựng nhu cầu tài chính vào ngày mai).
Anything else?
Paola Rossi (11:18 A.M.) That’s it. Depending on how much we finish, we could
develop a
timeline for finalizing the report this week.

65. For what aspect of the project is Mr. Lizzardi most likely responsible?
(Lizzardi phụ trách
mảng nào của dự án?)
(A) Production
(B) Budget
(C) Technology
(D) Personnel
66. Why does Mr. Lizzardi decline to help out? (Tại sao Lizzardi từ chối giúp đỡ?)
(A) He does not have time to work with Ms. Rossi.
(B) He has to attend a technology meeting.
(C) He cannot access the budget.
(D) He does not have information on participants (Ông ấy k có thông tin về người
tham dự)
67. When does Mr. Durav suggest meeting?
(A) On Monday
(B) On Tuesday
(C) On Wednesday
(D) On Thursday
68. At 11:18 A.M., what does Ms. Rossi most likely mean when she writes, “That’s
it”? (Rossi ý
nói gì khi viết “That’s it.”
(A) She does not need any other help. (Cô ấy k cần giúp đỡ gì thêm)
(B) She found the file she needs.
(C) She selected a new team logo.
(D) She has finished the end-of-year report.

Questions 69-72 refer to the following article.


Iwoni Media in the News
TORONTO (1 May) - Publisher Iwoni Media announced on Friday morning that new issues
of
Energy Run would not be released according to the book’s regular monthly publishing
schedule
(Công ty PIM thông báo hôm sáng thứ 6 rằng số mới của Tạp chí Energy Run sẽ không
được phát
hành vì kế hoach xuất bản sách của tháng) . — [1] —.
The company’s decision is connected to plans to modernize its printing facility by
replacing the
now-obsolete machinery on which the legendary comic book is printed. — [2] —.
As the word spread on Friday of the decision to halt publication, fans rushed to
buy what many
feared might be the last issue (Người hâm mộ đổ xô đi mua sách mà họ nghĩ là không
xuất bản
nữa). — [3] —. Iwoni Media’s online store is reportedly the only place where the
publication is
still available.
Iwoni Media’s spokesperson, Jon Emanuel, stated that the new printing facility
should be up and
running within four months and that the next issue of Energy Run will arrive on
newsstands shortly
after that (Trang thiết bị in mới sẽ hoàn tất và hoạn động trong 4 tháng nữa và số
mới của Enegry
Run sẽ có mặt tại sạp báo ngay sau đó). — [4] —.
“Fans should not worry,” Mr. Emanuel said. “Their favourite characters will be back
soon.”

69. What is the purpose of the article?


(A) To summarize the plot of a popular comic book
(B) To publicize the opening of an online store
(C) To report on a company’s recent earnings
(D) To describe a change in a company’s operations (Miêu tả sự thay đổi hoạt động
của công ty)
70. What is suggested about sales of Energy Run? Điều nào nói về doanh thu của
Energy Run?
(A) They increased suddenly. (Doanh thu tăng bất ngờ)
(B) They will be reported online.
(C) They were expected to be low.
(D) They were calculated incorrectly.
71. What did Mr. Emanuel announce? (Emanuel thông báo gì?)
(A) A new character will be introduced.
(B) A publication will become available again. (Sách lại tiếp tục được xuất bản)
(C) Some newsstands will be closed.
(D) Some book prices will be reduced.
72. In which of the positions marked [1], [2], [3], and [4] does the following
sentence best belong?
“To that end, the company has invested $100,000 in new equipment.” (Rốt cuộc thì,
công ty đã đầu
tư 100,000 đô la mua thiết bị mới.)
(A) [1]
(B) [2]
(C) [3]
(D) [4]

Questions 73-75 refer to the following notice.


Dear Customers,
After 45 years in business, Ghearey Garden Goods locked its doors for the final
time on Saturday,
January 10. As many of you know, I purchased a farmhouse in the country several
years ago, and I
had been driving three hours a day to get to the store and back (Mỗi ngày lái xe 3
tiếng đi đến của
hàng và quay về). While I am saddened to walk away from this business, it is just
no longer
sustainable for me to spend so much time in transit.
My grandfather, Timothy Ghearey, who built the business nearly 60 years ago before
handing it
off to me, always said that his favorite part about running a retail store was
serving the needs of his
community. I echo this sentiment. Words cannot express how much joy I have received
from
interacting with all of you, and I want you to know how much I value your dedicated
support over
all these years.
Thank you for understanding what a truly difficult decision this was. But while the
physical
Ghearey Garden Goods location is closing, a ghearygardengoods.com will remain
resource for
home gardeners through our Web site. I will keep adding educational blog posts and
articles as I
have time. (Chúng tôi sẽ tiếp tục thêm những bài viết mạng vào báo chí dạy làm
vườn.)
Thank you for your support!
Jerry Sanderson
Jerry Sanderson
Ghearey Garden Goods
73. Why did Ghearey Garden Goods close? Tại sao Ghearey Garden Goods lại đóng cửa?
(A) The building is being renovated.
(B) The rent costs became too expensive.
(C) The commute was too long for the owner. (Quãng đường đi làm quá xa đối với ông
chủ)
(D) An open management position could not be filled.
74. What does Mr. Sanderson indicate in the notice? Sanderson nói gì trong thông
báo?
(A) New competitors have entered the marketplace.
(B) The business was founded by a family member. (Doanh nghiệp này đc thành lập bởi
1 thành
viên trong gia đình)
(C) The store’s staff members are reliable.
(D) The demand for gardening supplies has increased.
75. What will people continue to find on the Web site? Trên trang web, khách hàng
có thể thấy gì?
(A) Photographs of a farmhouse
(B) Discount coupons
(C) Instructions for gardeners (Hướng dẫn người làm vườn)
(D) A list of recommended stores

Questions 76-77 refer to the following article.


CHICAGO (January 8)—Local retailer Derbyshire Company, which sells clothes,
household
goods, and other items through its twelve Illinois stores, announced today that it
will be buying the
popular bookseller Lillard, Inc (Thông báo rằng nó sẽ mua ại Công ty Lillard, là
một công xuất
banrnhieeuf sách bán chạy). The two companies will remain independent of each other
until April
30, when all Lillard’s merchandise—books, music, and other media—will be
transferred to the
Derbyshire Company stores.

Derbyshire Company CEO, Cynthia Schulman, stated that her company “could not be
more
pleased to bring customers the expanded selection of merchandise gained from this
merger with
Lillard, Inc.”

Meanwhile, Lillard’s president Steven Paulson announced that “from our point of
view, this was a
perfect opportunity for collaboration.” Paulson will stay on after the acquisition
with a job title yet
to be announced (P sẽ tiếp tục ở lại sau sáp nhập với vị trí mới chưa được công
bố). The companies
said they began talks eleven months ago but waited until the new year to announce
the news.

76. What is the purpose of the article?


(A) To announce the selection of a new CEO at Derbyshire Company
(B) To describe events at a store’s anniversary
(C) To notify the public of a business acquisition (Thông báo cho công chúng về
việc mua lại 1
doanh nghiệp)
(D) To attract applicants for job openings
77. What is indicated about Mr. Paulson? (Điều nào đúng về ông Paulson?)
(A) He is taking on a role at Derbyshire Company. (Ôn sẽ đảm nhiệm chức vụ tại công
ty
Derbyshire Company.)
(B) He is retiring from Lillard, Inc.
(C) He is starting a new company.
(D) He is moving to Chicago.

Questions 78-81 refer to the following notice


Welcome to the Smythe and Lewes team! We look forward to helping you build a career
with us.
We pride ourselves on the professionalism of our employees. Therefore, we offer the
following
tips to help you serve customers better and make your work as productive as
possible.

We specialize in (chuyên về) well-made formal and business attire for men and women
from
respected manufactures. We expect employees to wear similar attire at work, and we
encourage
you to wear products from our stores. Therefore, we offer you a 40% discount on all
merchandise
including shoes and accessories at all Smythe and Lewes locations (Do vậy, chúng
tôi tặng cho bạn
mức giảm giá 40% cho tất cả hàng hóa bao gồm giáy và phụ kiện tại tất cả các cửa
hàng của
Smythe and Lewes) . This will allow you to promote our store and, at the same time,
to develop a
professional wardrobe of your own.

It will also introduce you to the products and fashions we carry. As you can
imagine, customers
expect Smythe and Lewes employees to be knowledgeable about our inventory (Như các
bạn biết,
khác hàng kỳ vọng nhân viên của S and L nắm được hàng tồn kho). Please make an
effort to
familiarize yourself with it (xin hãy làm quen với việc đó). This task is best
reserved for periods
when there are fewer customers in the store- in the morning for weekday shifts and,
for weekend
shifts, at night before closing. As our inventory changes from week to week, this
needs to be an
ongoing process. With a little effort, you will soon be able to answer questions
from our customers
confidently!

78. For whom is the notice intended? (Thông báo này dành cho ai?)
(A) Clothing manufactures
(B) Smythe and Lewes customers
(C) Newly hired employees (Những nhân viên mới được tuyển dụng)
(D) Smythe and Lewes executives
79. What is probably NOT sold at Smythe and Lewes store? (Có lẽ hàng hóa nào k đc
bán tại
Smythe and Lewes?)
(A) Men’s accessories
(B) Business suits
(C) Formal shoes
(D) Athletic apparel (Trang thiết bị thi đấu điền kinh)
80. What will Smythe and Lewes give to recipients of the notice? (Smythe and Lewes
tặng gì cho
người nhận thông báo?)
(A) A discount on store products (Giảm giá cho tất cả các sản phẩm)
(B) Sample items from manufacturers
(C) Two breaks during every shift
(D) Free delivery on large orders
81. What are recipients encouraged to do? (Người nhận thông báo được khuyến khích
làm gì?)
(A) Avoid wearing business attire
(B) Study the store’s inventory (Tìm hiểu về hàng tồn kho)
(C) Return defective products
(D) Replenish the stock regularly

READING TEST 2. ONLINE


In the Reading test, you will read a variety of texts and answer several different
types of reading
comprehension questions. The entire Reading test will last 75 minutes. There are
three parts, and
directions are given for each part. You are encouraged to answer as many questions
as possible
within the time allowed.
You must mark your answers on the separate answer sheet. Do not write your answers
in your test
book.
Part 5
Directions: A word or phrase is missing in each of the sentences below. Four answer
choices are
given below each sentence. Select the best answer to complete the sentence. Then
mark the letter
(A), (B), (C), or (D) on your answer sheet.

1. _________ is currently Ms. Wonderle’s first year working as our office’s


administrative
assistant.
(A) This
(B) That
(C) These
(D) Those
2. Riversedge City is ___Adj (V /V )______ to announce new monthly parking
discounts for its
ed ing

residents.
(A) pleasing
(B) pleasure
(C) please
(D) pleased (vui mừng/ be pleased to do ST: vui mừng làm chuyện gì)
3. Sang-Joon Park is retiring after 25 years of ___N______ with Dahl Legal
Consultants.
(A) service (dịch vụ/phục vụ)
(B) profession
(C) knowledge
(D) relationship
4. The Trails store will ___Adv______ add a section for hiking equipment.
(A) eventualities (n)
(B) eventual (adj)
(C) eventually (adv) cuối cùng
(D) eventuality (n)
5. Puraforce staffing can provide your business ____with _____ temporary employees
during the
busy holiday season.
(A) from
(B) with (provide SB with ST: cung cấp cho ai cái gì)
(C) about
(D) into
6. In contrast to the _____N____ outside, the inside of Taft’s Bakery was warm and
welcoming.
(A) to weather
(B) weatherability
(C) weathering
(D) weather (n)
7. Berrell Foods, a wholesale distributor of fresh produce (công ty bán sỉ nông sản
tươi),
____V_____ more than 100 grocery stores (tiệm tạp hóa) in northern Scotland.
(A) contracts
(B) affords
(C) supplies (cung ứng/ cung cấp)
(D) travels
8. Payments to the Wendell Lake Association will be considered late _________ they
are received
after January 1.
(A) so
(B) by
(C) to
(D) if (câu điều kiện loại 1)
9. Although the alternate route (tuyến đường thay thế) was _____Adv____ marked
(đánh dấu),
many drivers ended up getting lost (lạc đường).
(A) never
(B) next
(C) clearly (rõ ràng)
(D) noisily
10. Later today (chút nữa), Mr. Warken _________ interview times for the job
applicant finalists.
(A) has been arranging
(B) will be arranging (sẽ sắp xếp)
(C) was arranged
(D) have arranged
11. For one week _________, Penny’s Grocery is giving away a free tote bag with
every order of
50 dollars or more.
(A) often
(B) only (chỉ)
(C) over
(D) through
12. Because of a recent artwork donation (quyên góp nghệ thuật gần đây), the
Libsing Art
Museum will soon be displaying a new __N(số ít)_______.
(A) exhibits
(B) exhibition (n) triển lãm
(C) exhibited
(D) exhibiting
13. Albert Doime oversaw smelting operations (giám sát kinh doanh) in Lydenburg for
a
___Adj______ time before being reassigned (phân công) to Johannesburg.
(A) brief (ngắn= short)
(B) large
(C) slow
(D) proper
14. The agreement ___Adv______ states (thỏa thuận tuyên bố cụ thể rằng) that Amy
Dyer will be
the general contractor (tổng thầu) for the Hibley project.
(A) specific (adj)
(B) specifically (adv: cụ thể)
(C) specifiable (adj)
(D) specified (v)
15. Executives (Quản lý) at Wess Lumber were praised (được khen) for addressing
employees’
concerns in a _____adj____ manner.
(A) constructing (v)
(B) construction (n)
(C) constructive (adj: có tính xây dựng)
(D) construct (v)
16. Flu season is here, so take advantage of the free flu shots (mũi tiêm cảm cúm
miễn phí) which
are being offered _________ in the lobby (sảnh).
(A) being offered (cụm phân tư rút gọn: đang được cung cấp)
(B) to offer
(C) offering
(D) offers
17. Clarkson Smith Legal Services requires that ___Adj______ client files be kept
in off-site
storage for seven years.
(A) misplaced
(B) inactive (ngưng hoạt động/cũ)
(C) unable
(D) resigned
18. By _____switching ____ to digital accounting (Bằng việc chuyển sang kế toán
điện tử), your
business can save time, space, and money.
(A) advising
(B) proving
(C) resolving
(D) switching (switch to: chuyển sang)
19. Mr. Liu’s long list of awards (danh sách dài phần thưởng ) can be attributed to
his skills
____as_____ a designer (kỹ năng làm thiết kế).
(A) on
(B) in
(C) as (làm/là)
(D) at
20. MBR Global Marketing has signed several new client contracts _which total
(totaling)
________ 12 million pounds in annual revenue (doanh thu của năm 12 triệu bảng).
(A) totaled
(B) total
(C) totals
(D) totaling (tổng cộng)
21. _________ teaching economics at the local university (ngoài dạy kinh tế học ở
một trường đh
tại địa phương), Ms. Ito also writes a monthly column (mục rao vặt) for a financial
magazine (tạp
chí tài chính).
(A) Besides (ngoài ….ra)
(B) Whereas
(C) Either
(D) How
22. The feasibility study (nghiên cứu tính khả thi) for building a new bridge (cầu)
was quite
complex and included several (=some: một vài)____N(số nhiều)_____.
(A) annotate
(B) annotative
(C) annotating
(D) annotations (dữ liệu)
23. The department mentor (giám sát của bộ phận) instructed the interns (hường dẫn
cho người
thực tập) to contact her first _________ they had any questions.
(A) in spite of (+N)
(B) as a result (adv)
(C) whenever (bất cứ khi nào)
(D) because
24. The number of people working at Yolen Laboratory’s two locations (địa điểm)
keeps
increasing, and there are now 452 employees ___adv______.
(A) apart
(B) enough
(C) yet
(D) altogether (in total: tổng cộng)
25. Highlee Sportswear, ____whose_____ popularity is widespread among athletes (độ
nổi tiếng
của nó ttrair khắp giới điền kinh), will add a line of children’s clothes soon (sẽ
sớm thêm dòng sản
phẩm thời trang thiếu nhi).
(A) whose (+N): đại từ liên hệ
(B) some
(C) major
(D) which
26. Analysis of the commercial building sector (Phân tích ngành xây dựng thương
mại) in Canada
has been ____Adv____ enhanced with the release of the Natural Resources Survey.
(A) greater (adj)
(B) great (adj)
(C) greatly (adv)
(D) greatness (n)
27. You are eligible (đủ điều kiện) either for a 15% discount off any ___Adj_____
price (giá tiêu
chuẩn) or for a 5% discount off any promotional price (giá khuyến mại).
(A) numerous
(B) duplicate
(C) standard (chuẩn)
(D) divided
28. ____Adj____ exercise is an important means (=cách) of improving health and
preventing
obesity (ngăn béo phì).
(A) Regular (adj: thường xuyên)
(B) Regularly (adv)
(C) Regularity (n)
(D) Regularize (v)
29. The insurance company said it needed a ____Adj____ report (báo cáo toàn diện)
on exactly
(chính xác) how the accident happened.
(A) complete (v, adj: toàn diện)
(B) completes (v)
(C) completing (v)
(D) completely (adv)
30. Mr. Kim, the plant manager (giám đốc nhà máy), expressed his support (bày tỏ sự
ủng hộ) for
____N/V _____ of a new parking lot (bãi đậu xe mới).
ing

(A) constructive (adj)


(B) construction (n): việc xây dựng
(C) constructively (adv)
(D) constructed (v)
31. Mr. Sam has been working _____as____ a marketing consultant (tư vấn marketing)
for the past
ten years.
(A) among
(B) as (work as = be: là/làm)
(C) behind
(D) within
32. The construction (xây dựng) of the new stadium (sân vận động) has been
postponed
(hoãn)________ the lack of funding (thiếu vốn).
(A) due to (+N: bởi vì)
(B) because (+ Cl: bởi vì)
(C) although
(D) if
33. Due to the most recent increase in freight rates (phí vận tải hàng hóa), we
find it __adj______
to raise our prices (cần thiết phải tăng giá) on Brazilian coffee.
(A) necessary (adj)
(B) necessarily (adv)
(C) necessity (n)
(D) necessitate (v)
34. It is not easy to get a table at a famous restaurant unless you call
__ahead______ for
reservations (gọi trước để đặt chỗ).
(A) above
(B) along
(C) ahead (=beforehands: trước/ call ahead: gọi trước)
(D) aside
35. Fifty countries around the world could be threatened (bị đe dọa) by rising sea
_levels (mực
nước biển dâng)_______ because of global warming.
(A) aptitudes
(B) rates
(C) balances
(D) levels (sea level: mực nước biển)
36. The cost of construction (giá xây dựng) as __well _____ as the construction
period (thời gian
xây dựng) has already increased twofold over previous estimates (báo giá ban đầu).
(A) good
(B) well (as well as = and: và, cũng như)
(C) fine
(D) far
37. Please __ V _____ of the Planning staff (Debra Ross @ 222.3926) if you
will or will not
(infinitive)

be attending this month’s meeting.


(A) notice
(B) present
(C) deliver
(D) notify (báo/thông báo= inform)
38. Most of the companies have a policy stating that each employee must ____ V
___ a

(infinitive)

formal, annual, and written performance appraisal (bản đánh giá kết quả làm việc)
by their
manager.
(A) give
(B) be given (được cho/cấp)
(C) be giving
(D) have given
39. Hotel guests (khách của khách sạn) who have some spare time (dư thời gian) are
____Adj
(V /V )___ to take part in the boat trip excursion to the nearby island.
ing ed

(A) encouraging
(B) encourage
(C) encouraged (được khuyến khích)
(D) encouragement
40. The government____Adv___ released the newest employment figures (công bố số
liệu việc
làm mới nhất) this morning.
(A) ever
(B) just (vừa mới)
(C) soon
(D) lately
Part 6
Directions: Read the texts that follow. A word, phrase, or sentence is missing in
parts of each text.
Four answer choices for each question are given below the text. Select the best
answer to complete
the text. Then mark the letter (A), (B), (C), or (D) on your answer sheet.

Questions 41-44 refer to the following notice.

Hollydale Medical Center


To All Office Staff:

The move to our new medical center building will take place this weekend.
(41)__Adv_____, all
items in your desks must be packed before the end of the day on Friday. The empty
boxes (thùng
rỗng) (42) ___V____ to you by 8:00 A.M. on Thursday. When you arrive (đến) at our
new
location (địa điểm) on Monday morning, your boxes should already be in your new
office. (43)
_______. If you will be out of the office this Thursday or Friday, notify me
immediately (báo cho
tôi ngay lập tức) so we can make other arrangements (sắp xếp khác).

We appreciate (trân trọng) your cooperation (hợp tác) in helping to make this
transition (chuyển
tiếp) to our new location go as (44) ___adv____ as possible (diễn ra suôn sẻ).

Jian Tang, Office Manager

41. (A) Therefore (adv: do đó) 43. (A) Enjoy dining at the nearby
cafeteria.
(B) Thirdly (adv) (B) You need to check your office mailbox.
(C) Regardless (adj) (C) The moving company has positive reviews.
(D) Instead (adv) (D) Please begin unpacking right away.(xin
hãy tháo dỡ ngay
lập tức)
42. (A) delivered
(B) have delivered 44. (A) directly
(C) will be delivered (sẽ được giao) (B) smoothly (suôn sẻ)
(D) had been delivered (C) slowly
(D) actively

Questions 45-48 refer to the following e-mail.


This year marks (đánh dấu) the twentieth anniversary (kỷ niệm) of Hispanic Heritage
Month.
(45)__Adv_____, the food court (khu ẩm thực) at the Swift Business Complex will
feature
cuisines from Hispanic and Latin American countries. Visit the food court today to
kick off the
(46)___N______ (tận hưởng tiệc tùng)! Free tapas (small bites) and drinks will be
available for
tasting.
(47)___Adv____, the Swift Business Complex (Khu phức hợp thương mại Swift) will be
hosting a
Hispanic Heritage Expo (Triển lãm di sản Tây ban nha) the week of October in the
center of the
main atrium. International businesses as well as local vendors will be
participating in this one-of-a-
kind event. Each afternoon at 3 P.M., two lucky shoppers will win leather handbags
from Cuero
Suave, a Colombia-based trading company located on the fourth floor (tầng thứ 4 của
tòa nhà).
(48)_______.

45. (A) To celebrate (v) 47. (A) Rather


Để kỷ niệm (B) However
(B) It celebrated (C) In addition (= and: và, ngoài ra, thêm vào đó)
(C) By celebrating (D) On the contrary (=however)
(D) The celebration
48. (A) The main atrium was completely renovated
last spring.
46. (A) festive (adj) (B) Visit the service desk to get your free ticket
for the drawing. (Đến
(B) festivities (n) quầy dịch vụ nhật phiếu miễn phí tham gia rút thăm
trúng thưởng)
(C) festively (adv) (C) Try the chicken tacos for a tasty treat.
(D) more festive (adj) (D) The food court will be open for breakfast all
week.

Questions 49-52 refer to the following e-mail.


To: Hye-Jin Lee
From: Globiance Technologies
Subject: Information
Date: 5 June

Dear Ms. Lee,


Thank you for (49)__V _____ the Sciorama X200 social science database (cơ sở dữ
liệu khoa học
-ing

xã hội Sciorama X200) . Your selection is a preferred one among many researchers
(nhà nghiên
cứu). We would like to learn about your (50)_N______ (trải nghiệm của bà) with this
database
through a five-minute survey. As a token of (51)___Adj____ appreciation (Sự biết
ơn), you will be
automatically entered (tự động thêm vào) into a drawing (rút thăm trúng thưởng) to
win a $100 gift
card from Globiance Technologies. All of your responses will be kept confidential
(Tất cả các câu
trả lời sẽ được giữ bí mật). (52)_______.
The questionnaire is available at www.gt.org/scioramasurvey. Thanks in advance for
your
feedback.

Sincerely,
The Survey Team at Globiance Technologies

49. (A) giving 51. (A) her


(B) performing (B) your
(C) writing (C) our (appreciation: sự biết ơn của chúng tôi)
(D) choosing (chọn (D) their
mua)
52. (A) They will be used for statistical purposes
only. (Chúng chỉ
50. (A) experience (v, n) được dử dụng vào mục đích thống kê)
(B) experiencing (v) (B) They will determine which solution will be
adopted.
(C) experienced (C) They will reveal what difficulties new hires
have reported.
(D) are experienced (D) They will be thoroughly reviewed for errors.
Part 7
Directions: In this part you will read a selection of texts, such as magazine and
newspaper articles,
e-mails, and instant messages. Each text or set of texts is followed by several
questions. Select the
best answer for each question and mark the letter (A), (B), (C), or (D) on your
answer sheet.
Questions 53-54 refer to the following sign.
HUMINI HARDWARE
Notice to Customers
Following the expansion of our store, and prompted by requests from customers for
more products,
many of the items (sản phẩm) between aisles (lối đi) 2 and 20 will now be relocated
(sắp xếp đến
chỗ mới). Reshelving will occur during the period of April 16 to May 2, at which
time we will
have additional staff on hand (có thêm nhân viên ) to help you find the products
you need (giúp tìm
ra sản phẩm khách hàng cần). We are confident you will find the new store layout an
improvement
over the old one.
Thank you for your patience.
Management, Humini Hardware

53. What is the purpose of the sign? (Mục đích của thông báo là gì?)
(A) To announce a temporary closure
(B) To advertise a new product line
(C) To explain how shelves are arranged
(D) To prepare customers for an upcoming change (Thông báo cho khách hàng về những
thay đổi
sắp diễn ra)
54. Why will extra staff be available? (Tại sao lại có thêm nhân viên?)
(A) To take inventory
(B) To help customers locate certain items (Giúp khách hàng tìm ra một số hàng hóa
nhất định)
(C) To give product demonstrations
(D) To help customers place orders

Questions 55-56 refer to the following Web page information.


https://www.rivieracarrentaỉ.co.uk/598270
Thank you, Mr. Gregersen!
Your Riviera Car Rental booking is now complete.

Today’s date: 3 June


Duration: 7 days
Pickup: 14 June in Bristol
Return: 20 June in Cambridge
Credit card: XXXX-XXXX-XXXX-2891
Amount: £310.00

You have opted to prepay your rental (chon trả trước tiền thuê), but your credit
card will not be
charged until 12 June. Until then you may cancel your booking at no charge (tận đến
lúc đó, ông
có thể hủy việc đặt thuê không tốn phí). Should you cancel after that date, a fee
will be charged.
Also, if you fail to pick up your car (no-show), you will be charged in full, and
the amount is
nonrefundable.

55. What is the purpose of the Web page information?


(A) To verify credit card information
(B) To approve a purchase order
(C) To describe accommodation options
(D) To confirm reservation details (Để xác nhận chi tiết về việc đặt thuê xe)
56. What is suggested about Mr. Gregersen? (Thông tin nào đúng về ông Gregersen?)
(A) His credit card needs to be replaced.
(B) His travel expenses will be reimbursed
(C) He can still cancel at no charge. (Ông vẫn có thể hủy đặt thuê k tốn phí.)
(D) He has requested a small car.

Questions 57-58 refer to the following article.


BAKERSVILLE (August 13)—As confirmed by today’s vote, the city’s building code is
getting a
shake-up. Among the changes to go into effect on November 3 are stricter fire
safety standards for
both commercial and residential buildings, enhanced insulation requirements, and,
most notably, a
requirement that 25 percent of newly constructed residential homes be equipped with
solar panels.

The solar initiative (cải tiến về năng lượng mặt trời) has been met with broad
support among
voters, lawmakers, and home builders. The new rule will increase the total amount
of green energy
produced and reduce dependency on fossil fuels. Supporters also say that increasing
demand for
solar panels and hardware will drive prices down, so that this technology will be
increasingly
affordable. Finally, though the building phase will be more costly than usual,
homeowners will
save on electricity bills in the long term.

The state will open a phone hotline to answer questions from builders, property
owners,
contractors, and other affected parties beginning on October 15 and continuing
through the end of
the year.

57. When does the new code become effective? (Khi nào luật/quy định mới có hiệu
lực?)
(A) On August 13
(B) On October 15
(C) On November 3
(D) On December 31
58. According to the article, what will become more expensive? (Theo bài báo, cái
gì sẽ trở nên đắt
đỏ hơn?)
(A) Construction costs
(B) The price of solar panels
(C) Home maintenance costs
(D) Household electricity bills

Questions 59-60 refer to the following chat discussion.


Axel Thorne (9:13 A.M.) Hi everyone. Our staff meeting scheduled for 3 P.M. today
has been
postponed until next week (Cuộc họp dự định lúc 3h chiều đã hoãn lại sang tuần
tới).
Beryl Smith (9:14 A.M.) Will it be at the same time next Tuesday?
Axel Thorne (9:15 A.M.) That's right, and the same place, in the conference room on
the first
floor.
Deanna Dahl (9:16 A.M.) I'm going to be on vacation next week. Could you send me
the meeting
minutes, please (Cậu có thể gửi cho tôi biên bản họp đc k)?
Axel Thorne (9:17 A.M.) No problem. I’ll take care of that. Enjoy your vacation.
Deanna Dahl (9:19 A.M.) Thanks! I'll review everything you talked about when I get
back.

59. What has changed about the meeting?


(A) The time
(B) The date (Thay đổi ngày)
(C) The location
(D) The topic
60. At 9:17 A.M., what does Mr. Thorne most likely mean when he writes, “No
problem”? (Lúc
9:17, Ông Thorne ý nói gì qua câu trả lời “No problem.”
(A) He will send Ms. Dahl a copy of notes about the meeting (Ông ấu sẽ gửi cho bà
Dahl ghi chú
của cuộc họp).
(B) He will ask someone to replace Ms. Dahl at the meeting.
(C) He will give Ms. Dahl a project to work on while she is away.
(D) He will meet with Ms. Dahl when she returns.

Questions 61-63 refer to the following instructions.


Skylight Gardens
Your Neighborhood Garden Center
Plant Care Tips
Effective plant care starts with choosing the right pot for your houseplant. Make
sure that there are
holes in the bottom of the pot to let water flow out.

Next, purchase soil from your local garden center. Store-bought potting soil
typically contains
fertilizer to help indoor plants grow (Đất để trồng trong chậu mua ở cửa hàng
thường chứa phân
bón giúp cây tăng trưởng). Using dirt from your own outdoor garden is risky. This
dirt can be
filled with insects, disease, and weeds, all of which can be harmful to indoor
plants.

After potting, pour water on the soil and flowers of your plant. Always check the
soil before
watering. If the soil is still moist, there is no need for more water.

To maintain a healthy plant, pinch or cut off parts of the plant that have turned
brown or withered
(Để cây khỏe mạnh, cắt tỉa những nhánh chuyển sang màu nâu hoặc héo). Doing so
helps to
facilitate new growth. Each spring, repot your plant in a larger pot to allow room
for the root
system to grow.

61. Why are readers advised to purchase soil (Tại sao độc giả được khuyên mua đất
sỏi?)?
(A) To avoid digging up their yards
(B) To promote plant growth (Giúp cây tăng trưởng)
(C) To support local garden centers
(D) To ensure that water is evenly distributed
62. What tip is NOT mentioned in the instructions? (Theo hướng dẫn, điều nào sau
đây k đúng?)
(A) Keep the plant in the sun (Đặt cây dưới ánh nắng).
(B) Replace the pot every year.
(C) Feel the soil before adding water.
(D) Use a pot with holes in it.
63. The word “turned” in paragraph 4, line 1, is closest in meaning to
(A) built
(B) rotated
(C) disturbed
(D) become (+ Adj= turn+ Adj: chuyển sang/ trở nên/trở thành; turn brown: chuyển
thành màu
nâu)
Questions 64-66 refer to the following notice.
Attention Conference Centre Staff:
The Geylang Conference Centre is hosting (đăng cai) the Singapore Banking
Conference on 20
April. The welcome desk will be open from 8:00 A.M. to 10:00 A.M. When checking in,
conference attendees will need to present a valid form of identification (Khi làm
thủ tục, người
tham dự phải trình thẻ chứng minh còn hiệu lực). Acceptable forms of identification
include a
passport, a driver’s license, or a company-issued photo ID. After checking in,
attendees will be
handed a welcome packet, which includes a conference schedule, a map of the
facility, and their
ticket to the evening banquet (phiếu tham dự tiệc buổi tối). If attendees arrive
after 10:00 A.M.,
they should be directed to the security desk, where someone will check them in.

Please note that some attendees will be staying at the conference centre’s hotel.
They should be
directed to the hotel lobby, where they can leave their luggage. Hotel personnel
will bring the
luggage to the appropriate guest rooms when the rooms are ready.

If you have any questions about your duties for the day, please contact Jia-Wei Teo
at extension
231 .

64. What is the purpose of the notice? (Mục đích của thông báo này là gì?)
(A) To inform staff about procedures for an event (Báo cho nhân viên quy
trình/trình tự của sự
kiện)
(B) To provide attendees with a schedule
(C) To ask for volunteers to work at an event
(D) To give information about conference speakers
65. The word “present” in paragraph 1, line 2, is closest in meaning to
(A) introduce
(B) show (trình ra/cho xem)
(C) attend
(D) gift
66. According to the notice, what will happen at the end of the conference? (Theo
thông báo,
chuyện gì sẽ diễn ra ở cuối của hội nghị?)
(A) Packets will be collected.
(B) Luggage will be put in storage.
(C) An evaluation form will be handed out.
(D) A banquet will be held (Tiệc sẽ được tổ chức).

Questions 67-69 refer to the following article.


GABORONE (6 May)—Local resident Sophie Shagwa has met a goal (mục tiêu) many
application
developers relish (=enjoy). — [1] —. Her app, Dream Sweet, which she created as a
participant in
the Next Generation Apps programme, has been downloaded more than 10,000 times.

“Simply put, this app helps users attain their dreams and ambitions, (Nói đơn giản
thì app này giúp
người dùng đạt được mục tiêu và tham vọng của mình)” Ms. Shagwa said. “The app does
this by
providing a series of questions to help users identify their goals and break them
down into
achievable parts.”

Recently, she added a much-requested calendar feature (Gần đây bà thêm tính năng
lên lịch mà
mọi người đều mong muốn). — [2] —. Daily reminders and encouraging text messages
are sent
around the times when certain parts of the goal should be accomplished.

Additionally, Ms. Shagwa designs calendars, notebooks, and posters with


inspirational phrases that
can be purchased through the app. — [3] —. “They’re lovely to look at and provide
visual
reminders that your goals are important and that you have the ability to achieve
them,” she said.
The app is free, but additional features, such as personalized coaching and
progress tracking, cost
extra. Ms. Shagwa uses some of her profits from the Dream Sweet app to sponsor new
app
developers who want to join the Next Generation Apps programme. — [4] —.

67. What does the article mainly discuss? (Bài báo chủ yếu nói về điều gì?)
(A) The recruitment of young people for a new program
(B) The profile of the creator of a piece of software (Thông tin về người vết ra
phần mềm)
(C) How apps and related products are priced
(D) Recent changes across the software industry
68. What does the Dream Sweet app help users to do? (Ứng dụng Dream Sweet giúp
người dùng
làm gi?)
(A) Reach their goals (Đạt được mục tiêu của mình)
(B) Create visual text messages
(C) Design artistic posters
(D) Sponsor new app developers
69. In which of the positions marked [1], [2], [3], and [4] does the following
sentence best belong?
“Users can now plan a specific timeline of actions in order to be successful.”
(Người dùng có thể
lên kế hochj cụ thể cho các hành động để thành công.)
(A) [1]
(B) [2]
(C) [3]
(D) [4]

Questions 70-73 refer to the following email.


To: elvin.williams@gxtinvestments.com
From: barry.robledo@gxtinvcstments.com
Date: May 17
Subject: 301 N. Powell
Attachment: Photos

Elvin,
I walked through the property at 301 N. Powell Avenue today. I was able to see the
reception area,
the offices, and the kitchen, but the utility closet in the west hallway was
locked. I am sure an
inspector will be able to verify that the furnace and electrical circuit board in
that space are in good
working order should we choose to proceed with a purchase.

The building has all the space we need to accommodate our clients, including a
large parking area.
However, there are some issues that we will need to address (Tuy nhiên còn một số
vấn đề chúng
ta cần giải quyết) if we move forward: the carpet in the reception area is
discolored; some floor
tiles in the east hallway are cracked (một số gạch nền bị nứt); and there is
chipped paint in the
conference room. The small water stain on the ceiling of the conference room may
indicate a leak
in the roof. I have attached photographs of the problem areas.

Given the low price of the property, I think we should consider this location. We
could update the
space and still come in under budget. Our current lease expires in three months, so
we will need to
make a decision quickly. Let me know what you think.

Sincerely,
Barry Robledo

70. Why did Mr. Robledo send the e-mail? (Tại sao Robledo lại gửi email?)
(A) To request the extension of a lease
(B) To suggest opening a branch office
(C) To support relocating a business (Giúp di dời công ty)
(D) To oppose the sale of a property
71. What was Mr. Robledo NOT able to view? (Ông Robledo không thấy d=gì?
(A) The reception area
(B) The kitchen
(C) The furnace (Không thấy lò sưởi)
(D) The conference room.
72. The word “address” in paragraph 2, line 2, is closest in meaning to
(A) attend to (chắm sóc, giải quyết)
(B) write on
(C) publicize
(D) place
73. What is one of the problems Mr. Robledo mentions? Ông Robledo đề cập vấn đề gì?
(A) Some tiles have been broken. (một số gạch nền bị bể/vỡ)
(B) A key has been misplaced.
(C) The parking area is too small.
(D) The location is inconvenient.

Questions 74-77 refer to the following online chat discussion.


Akio Nishi [3:35 P.M.]: Did you both see the e-mail from Barbara with furniture
options for
the lobby (Các bạn đã đọc email Barbara gửi về việc lựa chọn đồ nội thất cho sảnh
hay chưa)?
Isidora Basque [3:37 P.M.]: Look at the brown leather chairs with wheels. They are
so much
more stylish than the current chairs.
Akio Nishi [3:38 P.M.]: They are attractive, but they don’t seem sturdy. What
do you think of
the big green ones with the plush seats?
Kriya Patel [3:39 P.M.]: I am opening it now. (Tôi mở email liền)
Isidora Basque [3:40 P.M.]: They look comfortable. But space is limited (nhưng
không gian thì

giới
hạn/nhỏ), and we need at least four chairs in
the
lobby. Those green chairs are really big.
Kriya Patel [3:42 P.M.]: Are you still looking at the Premium Office section?
Akio Nishi [3:43 P.M.]: No, we’re discussing the Budget Office options now.
Kriya Patel [3:44 P.M.]: Oh, good. Those choices are more affordable.
Akio Nishi [3:46 P.M.]: We should probably select a reception desk before the
chairs. Do
you like the second one, the yellow desk? I do.
Kriya Patel [3:47 P.M.]: I think it would fit nicely in that space. It is the
same size as the
current desk.
Isidora Basque [3:48 P.M.]: I think it’s perfect. I’ll tell Barbara now that we all
like that desk
(Tôi sẽ nói
với
Barbar rằng tất cả chúng ta đều thích cái bàn đó). Let’s
touch
base about the chairs after we have had the
chance to look at
all the choices.

74. What is mainly being discussed? (Người ta đang bàn luận về điều gì?)
(A) A new office location
(B) A decorating budget
(C) An interior design firm
(D) New furniture choices (Lựa chọn đồ đạc nội thất mới)
75. At 3:39 P.M., what does Ms. Patel most likely mean when she writes, “I am
opening it now”?
(Patel ý nói gì qua câu “Tôi mở nó liền.”
(A) She is going to read a recent e-mail. (Cô ấy sẽ đọc email)
(B) She is looking in a desk drawer.
(C) She is unwrapping a package.
(D) She is preparing to eat lunch.
76. What is suggested about the lobby? (Điều gì đc nói về sảnh?)
(A) It currently has large furniture.
(B) It does not have a lot of space. (Sảnh không có nhiều không gian)
(C) It is located on the second floor.
(D) It is stylishly decorated.
77. What will Ms. Basque most likely do next? Bà Basque sẽ làm gì kế tiếp?
(A) Place an online order
(B) Distribute a catalog
(C) Contact a coworker (liên lạc với đồng nghiệp)
(D) Hire a receptionist

Questions 78-81 refer to the following newspaper report.

Beginning tomorrow, there will be a number of changes in the bus schedule. These
changes are
temporary and are being made on account of the road construction that is going on
downtown. The
city hopes that these changes will only last a week (Thành phố hy vọng rằng những
thay đổi này
chỉ kéo dài 1 tuần), but the damage from the winter snowstorms was severe (Nhưng
thiệt hại do
bão tuyết vào mùa đông ra nghiêm trọng), so construction workers are not sure when
they will
finish. Further updates will follow. Here are the bus schedule changes:
• Bus number 3 will no longer go down Main Street. It will run on First Street. The
rest of its route
remains the same.
• Bus number 5 is cancelled until construction is complete (Xe buýt số 5 bị hủy cho
đến khi xây
dựng xong).
• Bus number 6 will turn left onto Rainbow Drive and then right onto Third Avenue.
• Bus number 10 will now stop at the Convention Center and City Hall (Xe buyst số
10 sẽ dừng tại
Convention Center and City Hall). It will also run every five minutes as opposed to
every fifteen
minutes.
These are the only changes being made. All other bus routes will be exactly the
same. Please pay
attention for more changes. The city will issue another report when the bus
schedules go back to
normal.

78. Which of the following buses is no longer running? Tuyến xe buýt nào sẽ ngưng?
(A) Bus 3
(B) Bus 5
(C) Bus 6
(D) Bus 10
79. Why is the city engaged in road construction now? Tại sao thành phố lại xây
dựng đường?
(A) A hurricane damaged the roads.
(B) There were too many thundershowers.
(C) There was too much snow in winter.
(D) The report does not give a reason.
80. How has bus number 10 changed? Xe buýt 10 thay đổi như thế nào?
(A) It runs less often now.
(B) It no longer goes down Main Street.
(C) It turns left on Rainbow Drive.
(D) It has added additional stops. (Sẽ dừng thêm ở một số điểm)
81. When does the city hope the schedule to go back to normal? Thành phố hy vọng
khi nào lịch
buýt trở lại bình thường?
(A) In three days
(B) By the weekend
(C) In one week (một tuần)
(D) After one month

You might also like